4R Solutions

February 11, 2018 | Author: rajatgoyal20 | Category: Petal, Flowers, Altruism, Confidentiality, Chimpanzee
Share Embed Donate


Short Description

4R Solutions...

Description

MCAT Practice Test IV

SOLUTIONS

IV

M CAT MEDICAL COLLEGE ADMISSION TEST

1

AAMC MCAT P RACTICE T EST IV S OLUTIONS Edited, produced, typeset, and illustrated by Steven A. Leduc National Director of Medical Research & Development, The Princeton Review

Special thanks to: Jennifer Wooddell Judene Wright

Copyright © 2001 by Princeton Review Management, L.L.C. All rights reserved. MCAT is a service mark of the Association of American Medical Colleges (AAMC). TPR is not affiliated with Princeton University. Version 1.0

www.PrincetonReview.com

MCAT P RACTICE T EST IV S OLUTIONS C ONTENTS : Verbal Reasoning .............. 3

Physical Sciences .......... 19 Biological Sciences ...... 28

010110

2

VERBAL REASONING Passage I 1. D Item I: Yes. The country’s level of development is included in a list of relevant factors given in lines 53-55. Item II. Yes. See line 53. “Circumstances” is a synonym for “context”. Item III. Yes. The author posits this as the most important consideration (lines 55-57). 2. B A: The passage never indicates who originated the concept of human rights, only that the idea was not forced on other nations by Western cultures. B: Yes. In the first paragraph, the author argues that the idea of human rights was familiar to most non-Western cultures before it was advocated to them by Western nations (lines 7-13). C: This choice contradicts the main idea of the first paragraph; that the idea of human rights is ancient and pervasive, and was not invented by certain Western cultures and then forced on the rest of the world. There is no mention of resistance to human rights specifically from non-Western cultures. D: The author states that they were familiar with the idea of freedom “even if there were no explicit covenants to that effect” (lines 10-12). “Traditional societies” here corresponds to the reference to non-Western cultures in line 9. 3. D A: This is the right answer to the wrong question. The claim that the idea of freedom was not alien to non-Western cultures comes later in the paragraph. The author cites the two documents mentioned in the question to show that the concept of human rights is an ancient one. B: Again, this is the right answer to the wrong question. It is true, according to the passage, but the author does not cite these two Declarations in order to show that it is true. Compare this choice to answer choice D. C: Again, right answer, wrong question. The second paragraph includes this statement in lines 14-15, but not in reference to the claim that human rights is an ancient idea as shown by the content of these two documents. D: Yes. This choice accurately describes the author’s purpose in this part of the passage. These two documents are cited as direct evidence that the concept of human rights is an ancient one. 4. A A: Yes. In lines 55-58, the author argues that the most important factor to consider is whether or not there is a trend toward greater human rights. The tone of the passage is strongly positive about increased recognition of human rights, and yet the author says that it will not happen all at once (lines 51-52). Thus, the author would call for cautious praise. B: While the passage does discuss monitoring in the third paragraph, the author never connects a positive trend towards recognition of human rights (as described in the question) with a need for intense monitoring (see lines 51-57). C: While the author argues that we must take level of development into account (lines 53-55), the passage never indicates that forgiveness of abuses is called for. In fact, the author states that there is no possible excuse for certain kinds of violations (lines 58-62). D: The passage states that implementation of human rights protections may take some time; we must consider whether or not there is a move toward human rights in a particular country when evaluating that case (lines 51-57). The scenario presented by the question clearly indicates a move in the right direction; the author would praise, not criticize this nation.

3

5. D A: In the passage as a whole, the author criticizes human rights abuses, and in the final paragraph sets out certain kinds of abuses that can never be tolerated, regardless of the country’s level of development (lines 58-62). However, the author never indicates that state use of private property qualifies as a human rights abuse at any level. Be careful not to use outside knowledge or personal opinion to answer questions. B: See the explanation for choice A. The author does not suggest that placing conditions (which could include age, citizenship, etc.) on the right to vote qualifies as a violation of human rights. C: See the explanation for choice A. The author does not argue that freedom can never be curtailed, even in emergency situations. D: Yes. In lines 58-61 the passage lists torture as one of several abuses that can never be tolerated, regardless of level of development. 6. C A: The author claims that the United Nations has failed to become “an effective instrument for the promotion of human rights” (lines 32-35). B: Always pay close attention to the word “most” (as in “most benefit”) when it appears in a question. While the Organization of American States did pass a human rights declaration (lines 37-39), the passage does not show that the OAS ever takes direct action to rectify specific abuses. Be careful not to use outside knowledge; the credited response must be directly supported by the passage. C: Yes. The author describes how Amnesty International calls public attention to specific abuses, and may “mobilize public support” to bring about action (lines 46-50). D: The author does not suggest that people being abused by their own government would benefit by appealing to the leaders of the abusive state. 7. B A: The correct answer will weaken or be inconsistent with a claim made by the author. While the passage never specifically calls for civil disobedience, the author does say that certain abuses are never tolerable (lines 58-62). We do not know that these “unjust laws” fall into this category, but the answer choice is consistent, not inconsistent with the overall tone of the passage. B: Yes. In the second paragraph the author asserts that human rights may transcend or supersede the laws of nations, in part through citing Dr. Evatt’s claims in lines 23-28. Thus the author argues that outside interference may in fact by called for, in opposition to the statement made in the answer choice. C: The passage itself suggests this to be true in its discussion of trends (lines 51-55). D: This choice is entirely consistent with the author’s statement that “the walls of oppression would not crumble at the first clarion call” (lines 51-52).

Passage II 8. A A: Yes. In paragraph 5, the author claims that “women writers,” by using that term to describe themselves, send the message that women need to segregate themselves only until they “become politically strong . . . a visible, viable social factor” (lines 48-51). B: This choice takes words from the passage out of context. The author asserts that women writers claim to recognize humanity-as-a-whole (lines 45-47), not that they are waiting for literature to recognize it. In fact, the passage indicates that true literature, by definition, recognizes the universality of humanity (lines 24-30). C: As in choice B, this answer takes words from the passage, but does not form them into an appropriate answer. The author states in lines 38-41 that feminists (who, according to the author are not “women writers”) believe that the imagination cannot be liberated, because it is already free. Women writers, as described in the passage, are not waiting for the liberation of the imagination, but for political and social power (lines 48-55). D: Again, this choice takes words and ideas from the passage out of the context of the question. The author refers to the appearance of a new generation of authors (lines 58-59), but this does not correspond to the time when women writers plan to rejoin the world (lines 48-55). 4

9. B A: This choice contradicts the passage. The author contrasts women writers who segregate themselves, believing that they have an inherent common ground as women (lines 15-21), with feminists who reject gender-based identities and selfsegregation (lines 31-40). Be careful to use the author’s own definition of feminism, as it is described in the passage, to answer the questions. B: Yes. According to the author, feminism came into being in order to eliminate the “myth-fed condition” of genderbased segregation (lines 32-34), and feminism in the arts arose to eliminate “mythological divisions” (lines 3840). C: This is what women writers (lines 14-23), not feminists do (lines 31-41). D: According to the passage, women writers already accept humanity-as-a-whole (lines 45-47). 10. C A: The author asserts that women who consciously write as women are engaged in the politics of sex (lines 1-3), not that all women who write use the language of politics. The author uses the label “women writers” to refer to something much more ideologically specific then simply “women who write.” B: According to the passage, the author believes that “the separate male and female states of intellect” do not in fact exist (lines 14-17, 24, 31-38). The author uses the term rather to describe the language of women who write as women, with a particular social and political agenda. The politics of sex may include a belief in separate intellects, but the author does not use the phrase to refer to that belief. C: Yes. In lines 8-13, the author describes contexts in which the politics of sex is appropriate, that is, in movements that fight in political or socioeconomic arenas for equality for women. The author objects to “women writers” on the basis that their writing assumes a certain agenda or set of premises (the politics of sex) from the beginning, rather than imaginatively seeking out new ideas (lines 14-23). Thus they import the politics of sex into literature, where the author believes it does not belong. D: The author does not believe any real separation exists. Furthermore, even women writers who engage in the politics of sex do not conceive of the separation as a permanent one (lines 55-56). 11. A A: Yes. The correct response will be a statement that is inconsistent with the author’s opinions as they are expressed in the passage. The author argues that the idea that women have separate life experiences is a myth (lines 31-38); throughout the passage the author asserts a commonality, not a distinction between men and women. B: Notice that this choice does not specify that these views would be expressed through literature. The author argues that literature [fiction and poetry (line 42)] should not be written in the language of politics (lines 12-14), but never indicates that female authors should not make their views known in other ways. C: This is a direct paraphrase of the passage (lines 24-26). The author would most likely agree, not disagree with his or her own words. D: The author mentions the struggle for equal employment as an appropriate arena for the politics of sex. The passage does not indicate that the author would oppose greater wage equality in any way; what the author criticizes is the incorporation of politics or politicized self-images into literature. 12. D A: The passage portrays the beliefs of women writers and feminists as mutually exclusive (lines 14-23, 31-40), and gives no hint that women writers are evolving into feminists. B: According to the author, the minute a person calls herself a woman writer, she becomes political (lines 1-7, 14-23). Thus, those who are called women writers do not eventually become political, but are already politicized. C: This choice is too extreme. While the author claims that the human component of literature is denied by women writers (lines 24-30), the passage does not suggest that that component will be eradicated from all literature if the label “women writer” is used. D: Yes. The author argues that the supposedly temporary strategy of segregation will turn into a permanent reality in 4-5 years, and authors will find themselves divided into two categories: “women writers” and (male) “writers” (lines 58-64).

5

13. D A: The correct answer will be a statement from the passage that is inconsistent with the scenario presented in the question. If the author did in fact admire the fiction or poetry of a woman writer, it would have no bearing on the passage’s discussion of the origins of classical feminism (lines 31-34). B: The author’s admiration for a woman writer’s work would not be inconsistent with the author’s depiction of the beliefs of women writers (lines 19-23). C: The scenario in the question is discrepant with the author’s disapproval of the work of women writers and concern over the potential impact of their use of the term “woman writer.” However, it is not specifically inconsistent with the author’s description of that impact [an uninspiring world in which all the “writers” are men (lines 60-66)]. Compare this choice with answer choice D. D: Yes. According to the author, women writers, by labeling themselves as such, inappropriately incorporate the language of politics into the world of literature (lines 12-14). Literature should be driven by the imagination and should build connections between the sexes. This it cannot do, according to the passage, if it is politicized (lines 24-30). Thus, if the author admired the work of a woman writer, it would be discrepant or inconsistent with the author’s assertion that the language of politics should not be used in literature. 14. A A: Yes. According to the passage, literature “engenders sympathies from sex to sex, from condition to condition...” (lines 24-28). B: The passage defines literature as fiction or poetry when the author defines a “writer” as a “fiction writer or a poet” in line 42 (see also lines 12-13, 24-30). This answer choice describes a nonfiction essay. C: While a novel is fiction, true literature according to the author foments sympathy, not antagonism between the sexes (lines 24-26). D: Such a tract would be non-fiction, and written in the language of politics. Both of these characteristics would disqualify it as literature in the eyes of the author (lines 12-13, 42).

Passage III 15. C A: The passage does not ever raise the issue of prisoners’ beliefs regarding their right to privacy. There is no direct evidence in the passage to support this choice. B: Prisoners’ concern or lack of concern for their own health is not an issue in the passage. C: Yes. The author argues that physician–patient confidentiality in prison is especially important, as patient distrust could cause prisoners to fail to disclose important health-related information (lines 13-15, 20-23). Thus it would be reasonable to conclude based on the passage that the inmates refused a diagnostic procedure because of this distrust. D: As in choices A and B, this choice requires too much speculation. While the author does raise the issue of detection of weapons (lines 38-43, 62-67), the passage does not suggest that prisoners refuse procedures out of fear that a weapon will be detected. Furthermore, the author does not indicate whether prisoners carry weapons for self-protection or for other, more aggressive reasons. Compare this choice to choice C—the passage directly addresses the possibility that patients may refuse to cooperate fully in their own diagnosis and treatment due to distrust of prison physicians. 16. D A, B, and C: All three of these choices are wrong for the same reason: they are not strong enough. The author places no conditions on or exceptions to the “duty to warn” held by a physician when a weapon is detected (lines 38-46). Later in the passage, the author does state that a patient should be given the opportunity to surrender the weapon voluntarily, but prison authorities are still involved (lines 62-67). D: Yes. In lines 38-46, the author asserts that the imminent threat posed by a weapon overrides the patient’s right to privacy, and the physician has a “duty to warn” the prison authorities.

6

17. D A: The passage indicates that a certain level of confidentiality and physician autonomy is necessary for good medical care (lines 9-15, 24-25). However, the author does not suggest that the absence of these qualities is the only possible cause of a decline in the quality of medicine. Therefore, if the quality declines, it is not necessarily because of a lack of confidentiality. B: The passage suggests that the ability to keep certain kinds of information from prison authorities is one factor in physician autonomy (lines 24-32). However, we don’t know from the passage that it is the only factor. Thus a physician could refuse to reveal information, and still lack autonomy. C: This choice is wrong for the same reasons as choice A. Respect for confidentiality is one necessary condition for good medical care (lines 9-15), but the passage does not claim that it is the only necessary condition. Therefore, confidentiality could be respected while the quality of care is poor for other reasons. D: Yes. The author claims that confidentiality and physician autonomy are necessary conditions for good medical care (lines 9-15, 24-25). Thus, if a physician was forced to break confidentiality, the quality of care would suffer. 18. B A: The author refers to “necessary information” in the context of explaining that without confidentiality, prisoners may fail to reveal information important for their own health care. The passage gives no indication that past criminal activities would likely be related to an inmate’s health. B: Yes. The author introduces the passage through the example of prisoner A, found by a physician to have drugs and paraphernalia on or in his person. The author then goes on to argue that information regarding drug use could be crucial to a patient’s treatment, so physicians should not be required to reveal this “necessary information” to the authorities (lines 13-23, 33-36, 67-70). C: The author uses the term “necessary information” to indicate information necessary to the patient’s medical treatment. Be careful not to take the words out of the context of the passage. D: Pay close attention to the words “most specifically” in the question. While one could imagine a patient’s psychiatric history being an important factor in their current medical treatment, psychiatric history is never specifically mentioned in the passage. However, the author explicitly and repeatedly argues that doctors must know of a patient’s drug use in order to effectively treat him or her (lines 1-7, 33-36, 67-70). 19. A A: Yes. By claiming that “respect for patient confidentiality is particularly important in a prison hospital setting” because patients might distrust prison doctors (lines 20-23), the author indicates that inmates are less likely to expect confidentiality than are non-incarcerated patients of private doctors. B: This choice takes words out of context of the passage. The author mentions incompetence as an exception to the requirement that informed consent be provided by patients (lines 56-59), not as a reason why confidentiality would be especially important. C: The question is not asking when an inmate’s ability to keep secrets is threatened. Confidentiality may not apply to cases where diagnostic tools reveal the presence of weapons, but physician confidentiality is not itself threatened by the use of these tools. Finally, the use of X rays is not depicted as “invasive” by the author. D: The author states just the opposite. While inmates do not have “full Constitutional rights to privacy,” physician confidentiality is one of those rights which they do have in most circumstances (lines 16-19).

7

20. B A: The correct response will be inconsistent with the author’s claim that physician confidentiality is especially important in prisons because inmates distrust doctors who are hired by the prison (lines 19-23). First of all, the author argues that confidentiality should not be respected when weapons are detected (lines 38-46). Secondly, prisoners’ awareness that X rays will detect weapons would not affect the author’s argument about the need for patients to be able to trust their doctors and the unlikelihood that inmates will do so. B: Yes. The author argues that inmate patients distrust prison hospitals because the doctors may be employees of the prison. This is the basis of the author’s assertion of the special need to respect physician confidentiality in prison settings. If inmates do not believe that prison doctors are controlled by the institution, then prison hospitals do not present a unique situation. Therefore, while the physician–patient covenant would still be important, it would have no special importance in a prison setting. C: This choice would strengthen, not weaken the author’s argument. If prison officials often question physicians about inmates, inmates would have reason to fear that information given to a physician about activities such as drug use might be revealed to the authorities. D: This choice is too vague to have any impact on the author’s argument. It does not specify whether or not prisoners misunderstand their Constitutional rights to privacy. Nor does it indicate whether this misunderstanding is of a sort that would decrease the trust they place in their doctors.

Passage IV 21. D A: This choice is too narrow to be the main idea. It is directly stated in lines 8-9, but it is only one small part of the author’s overall argument that altruism may be as “biological” as selfishness. B: This choice (directly stated in lines 20-22) ignores the final three paragraphs of the passage. The author goes on to argue that natural selection may allow for altruism as well through kin selection. C: While this is an important part of the author’s argument about altruism in animals (lines 32-33), the author goes on to claim that human altruism may therefore be just as “animal” or biological as selfishness. Thus it is evidence for the main point, not the main point itself. D: Yes. The author argues that animals act altruistically when those acts benefit kin. Therefore, human kindness may not be a “unique overlay,” but instead a characteristic that we share with the animal kingdom. 22. A A: Yes. In the first paragraph, the author argues that we tend to attribute selfish acts to our animal nature. We also tend to believe that we act for the benefit of others only when we can overcome that animal part of ourselves through our uniquely human capacity for reason or rationality (lines 11-14). Thus, according to common belief, altruism is a uniquely human imposition or overlay on our basic animal nature. B: This is the opposite of the correct response. In the context of the passage, the “unique overlay” is human rationality, which acts against our animalistic tendencies. C: As in choice B, this is the opposite of the correct response. The passage describes selfishness as an animalistic, not uniquely human behavior (lines 8-11). D: This choice takes the passage’s reference to self-criticism out of context. We criticize ourselves for “animalistic” behavior (lines 8-11), but the passage does not claim that the ability to do so is uniquely human. Be careful not to use common sense or outside knowledge. When the passage uses the term “unique overlay,” it is specifically in reference to altruistic behavior that we (falsely) believe to be non-animalistic. 23. C A: Altruism and rationality are not compared to each other. The passage states that we tend to believe that it is our rationality that allows us to behave altruistically (lines 11-14). B: Both traits are listed as part of our “apish ancestry” (lines 9-11), but selfishness and aggressiveness are not compared to each other. C: Yes. A major theme of the passage is the comparison between the belief that altruism is unique to humans (lines 8-14) and the author’s belief that animals as well may act altruistically through kin selection (lines 29-34, 53-56). D: Determinism is discussed in lines 44-52, while rationality is mentioned in line 13 in a very different context; the two are never compared to each other. 8

24. C A: The sentence given in the question refers to the belief that specific behavior (hoarding and giving) can be determined by (“attributed to”) genetics. This is just the opposite of free will as it is described in the passage (lines 46-50). B: The main point of the fourth paragraph (lines 51-66) is that altruistic behavior may have a biological basis, but is not completely determined by genetics. The passage discusses rationality in the first paragraph in a different context, when it describes the view of many that human beings use their unique capacity for reason to override animalistic, selfish impulses (lines 9-14). C: Yes. The section of the passage referred to by the question presents the author’s claim that specific behaviors are not completely determined by genetics. The statement in the question, that some have attributed both selfish and altruistic behaviors to genetics, could be used to illustrate “deterministic speculation” (lines 44-46). D: The author argues that human nature is not completely determined by genetics (lines 44-50). Thus a statement describing deterministic beliefs would not illuminate or illustrate that claim. 25. A A: Yes. This assertion is made in lines 46-50, with no reference to authority or example. “Free will” is described, but the passage does not provide an example of how free will affects human behavior. B: The author refers to Freud as an authority who has made this claim (lines 56-59), and gives Copernicus, Newton, and Freud as examples (lines 59-63). C: The passage cites Hamilton as an authority on this point (lines 26-32). D: The author supports this assertion through reference to Freud (lines 1-6). 26. B A: The author discusses natural selection in the second paragraph in order to introduce the question of how altruism could be biological (lines 19-25). The author does not deal with that question until the subsequent paragraph, through the discussion of kin selection. B: Yes. The passage discusses kin selection (lines 26-34) in order to resolve the question of how altruism could have a biological basis. C: The passage states that civilization requires altruistic behavior and the suppression of selfish and aggressive biological instincts (lines 1-6). That part of the passage does not deal with the paradox raised in the quoted section of how this altruistic behavior could have a biological basis; that resolution does not come until the author’s discussion of kin selection in the third paragraph. D: The question asks where the author most directly deals with the question of biologically-based altruism. Evolution is related to kin selection by the passage (lines 29-34), but it is in the discussion of kin selection itself that the author most directly deals with the problem. Compare this choice to answer choice B. 27. D A: The author indicates that while we once believed that we lived at the hub of the universe, we have been forced to accept that that belief is not valid (lines 56-60). B: The passage states that “before Freud, we imagined ourselves as rational creatures” (lines 61-63), suggesting that we no longer, post-Freud, see ourselves as rational. C: This choice is inconsistent with the author’s rejection of genetic determinism (lines 44-50). D: Yes. According to the passage, we once believed that our capacity for altruism was not based in biology, and so that our ability to be kind distinguished us from non-human animals (lines 11-14). However, once we recognize that altruism, like selfishness, may have a genetic, evolutionary basis, we may see that we have more in common with other animals than we thought (lines 53-56, 63-66). 28. A A: Yes. The main idea of the passage is that while we by nature are selfish and aggressive (lines 3-11), we are also by nature kind and altruistic (lines 42-44, 53-56). Thus the author would likely argue that our natural tendency towards conflict may be tempered or counteracted by our coexisting natural tendency towards kindness. B: This choice misrepresents the author’s position. The author argues that our true biological nature includes both brutality and altruism (lines 37-39, 42-44). C: The author claims that our biological nature includes both brutality and kindness (lines 37-39, 42-44). Furthermore, the passage never raises the issue of spirituality. D: The author introduces this view in the first paragraph in order to go on to reject it. The passage indicates that altruism is not imposed upon our biological nature by rationality, but may in fact arise from our biological nature (lines 26-42). 9

29. B A: The author rejects biological determinism (lines 44-50). B: Yes. The passage asserts that human social life and the survival of civilization requires reciprocal altruistic behavior; that is, that we all act for the good of others and of the social body (lines 1-6). C: The passage argues just the opposite: that in order to live together and ensure the survival of our civilization, we must temper or reduce our animal aggression through kindness and altruism (lines 1-6). D: The passage never mentions genetic mutation. 30. C A: This statement strengthens Freud’s argument by reinforcing his claim that we have a natural tendency toward aggression. B: This choice also strengthens Freud’s claim that our biology drives us towards animalistic, selfish behavior. C: Yes. If lower animals are capable of altruism as well as selfishness, then we may be able to behave altruistically not by suppressing but by embracing our “animalistic, biological instincts.” Thus this statement weakens Freud’s argument. D: The agonizing dilemma mentioned in the passage (with reference to Freud) is that we are selfish by nature, and yet civilization requires altruism (lines 1-6). This is not inconsistent with the statement by Freud cited in the question that we must renounce our nature in order to be altruistic.

Passage V 31. A A: Yes. The passage states that once beetles feeding on sap exuded by ovules (ovules play a vital role in reproduction) became a vehicle for pollination, the ovules themselves had to be protected (lines 24-26). B: This choice reverses the relationship between insects and flowers as it is presented in the passage. The author mentions a variety of ways in which flowers evolved so as to take better advantage of insects (lines 21-23, 24-28, 42-50, 53-65). The passage never indicates that flowers influenced the evolution of insects. C: The passage does discuss the dispersal of pollen to and from seed-bearing plants in the first two paragraphs (although the range of pollen dispersal is not discussed). However, dispersal of the seeds is never mentioned. Be careful not to rely on outside knowledge. D: The passage indicates just the opposite. Restriction, not expansion of the variety of means of fertilization, was the problem to be solved (lines 56-68). The passage does discuss the advantages of insect over wind pollination, but insects represented an alternative, not an additional means for individual plant species (lines 18-23). 32. B A: The passage refers to the structure and behavior of insects living 70-150 million years ago. Modern insect species never explicitly come into the picture. B: Yes. The tone of the passage is clearly speculative. The author constantly uses words and phrases like “probably” (lines 3, 6) and “must have,” “would have,” “may have,” or “could be” (lines 13, 17, 19, 29, 32, 48, 51). As for the theoretical basis of the passage, the author does not argue for or defend natural selection as an appropriate theoretical model, but simply assumes that the reader will accept it as valid. C: The only reference to modern times is a brief mention of modern survivors of ancient plants (lines 8-10). The passage is not framed in terms of (nor does it even discuss) ongoing evolutionary trends, however. D: Only once does the passage specifically mention the fossil record (lines 1-3). While the author may well be drawing on fossil evidence for his or her conclusions, the passage is primarily characterized by the author’s educated guesses about the likely course of flower evolution (see the explanation for choice B).

10

33. C A: In terms of number of species, the passage suggests just the opposite. Flowers evolved ways to screen out unwanted visitors and to restrict pollination to certain insect species (lines 65-68). In terms of sheer number of insects, the passage does not mention evolutionary trends that might have boosted the number of insects (regardless of species) attracted to flowers. By discussing a shift from “promiscuous” to “restricted” pollination, the passage indicates that this was not in fact the case. B: The passage states just the opposite. Reproductive structures evolved that screened out short-tongued, non-flying insects (see last paragraph). Furthermore, the author does not suggest that tubular corollas were more accommodating to “specialized and flower-constant species,” just that they were less accommodating to beetles and inadequately equipped flies. C: Yes. The flower petals fused to form a tubular corolla, which kept the attractive nectar hidden from and out of reach of beetles and short-tongued flies (lines 56-64). D: No mention is made of how the reproductive structure of flowers affected insect evolution. It may well have done so in reality, but it is never discussed in the passage. 34. C A: The correct answer will be related to a theme already present in the passage, as well as to the main idea of the passage as a whole. The passage is about the evolutionary influence of insects on flowers, and gives no hint as to what role climate might play. B: The passage presents the “big picture” of long-term evolutionary trends in flower structures. There is nothing in that picture that could be directly used to answer questions about biochemistry. C: Yes. The author speculates about how the structure of flowers, including pigmented petals (lines 48-50), may have evolved so as to maximize pollination by the most appropriate insects. This change, according to the author, contributed to reproductive success. This could be appropriately extended to speculation about how the color of flowers may play a role in attracting those same insects. D: We know from the passage when flowers originated (lines 1-3) and when specific types of winged insects (bees, moths, butterflies, and long-tongued flies) came into being (lines 53-55). We can infer from the latter discussion that other types of winged insects (including small flies, lines 60-62) existed before bees et. al. appeared on the scene, but the passage gives no indication of how long before. 35. B A: The correct answer will weaken the author’s assertion that corollas evolved in order to restrict access to certain insects that fly regularly between flowers of the same species (lines 60-68). This choice would tend to strengthen that claim by suggesting that “promiscuous pollination” was now no longer possible, and that these flowers now depended on a limited number of insects pollinators. B: Yes. If the long-tongued insects of the Tertiary Period (lines 53-57) could not fly, then the author’s hypothesis that the corolla evolved in the Tertiary Period in order to allow access only to long-tongued flying insects (lines 61-64) is questionable. C: The fact that the genes which control one advantageous development (corollas) also cause a disadvantageous development (fungus) would have no impact on the author’s claims about why and how insects are related to the evolution of flowers. D: Whether or not flowers can now be chemically induced to form corollas has no bearing on the author’s argument about how corollas originally evolved. 36. D A: To answer the question, one must frame the botanist’s claims in terms of the issues raised in the passage. The passage concerns itself with the evolution of insects only to the extent that it affects the evolution of flowers. Knowing why beetles retained short tongues would not help us understand why flowers evolved to exclude them. B: The botanist, in the context of the passage, would need to explain why flowers evolved to admit only long-tongued flies. The evolutionary goings-on of bees, moths, and butterflies has no direct relevance to this question. C: We don’t care how the flies survived before corollas evolved. We care why the flowers evolved so as to attract only these flies. D: Yes. In the passage, the author describes restrictive pollination as a “great step forward in floral design” (lines 65-68), and throughout the passage refers to advantages conferred by various stages of floral evolution. Thus, to explain in the context of the passage why flowers evolved to be pollinated by a single species of fly, the botanist would need to explain why this represents a reproductive or evolutionary advantage. 11

Passage VI 37. D A: This statement, while supported by the passage (lines 4-6), is too narrow to be the central thesis of the passage. The author uses orangutans as one example in the context of the passage’s overall argument about training and sensitivity. B: The author writes that orangutans are different than chimpanzees (paragraphs 2 and 3), but does not suggest that this difference makes them more interesting. In fact, the passage states that chimps look “inferential, ingenious, and ever so active” while problem solving (lines 15-16), which sounds like reasonably interesting behavior. Even if this choice were supported by the passage, it would be too narrow to qualify as the main idea or central thesis. C: This choice takes words from the passage out of context. The final paragraph argues that a better understanding of animals may help us better respect other species, not our own. D: Yes. The first three paragraphs demonstrate the intelligence and responsiveness of animals. The final two paragraphs build on that theme to argue that “true training” must be reciprocal, which requires sensitivity to and respect for these qualities in the “trainee.” 38. C A: The passage does not discuss the comparative speed with which chimps and orangutans understand or solve problems. B: Chimpanzees use a different method to solve problems, but the passage does not indicate that it is a more appropriate method. C: Yes. The passage describes how the chimpanzee takes the hexagonal peg and experiments until it finds the correct hole (lines 15-21). The orangutan, on the other hand, appears to be uninvolved in the test until it “casually, as if thinking of something else” completes the task (lines 22-29). Thus we cannot observe how the orangutan has solved the problem, while the chimpanzee looks “inferential, ingenious and ever so active.” D: The author cites Prof. Miles’ judgment that orangutans are no less smart than chimpanzees (lines 29-31) and her evaluation of the nature of their intelligence (lines 31-34). 39. D A: While the author claims that orangutans have few social encounters (lines 6-10), the passage never indicates whether or not they are aggressive in the encounters that they do have. B: A lack of social interaction does not constitute or indicate a lack of responsiveness to any external stimuli. In fact, the passage shows in its discussion of Berosini’s orangutans that they can be trained (lines 1-4, 35-38), and in the third paragraph describes how orangutans do respond, in their own way, to the stimulus provided by the intelligence test. C: While orangutans may be difficult to train because they “are not dependent on social support and approval” (lines 410), the author does not describe this as “temperamental” behavior. D: Yes. While we might not naturally think of the word “poetic” in order to describe independent behavior, that is the way in which the author uses the term in the context of the passage. In the first paragraph, the passage explains how orangutans have little social interaction with each other in the wild. The quote from Miles is given to illustrate the claim that an orangutan is “irredeemably his or her own person,” that is, not interested in the support and approval of others (lines 8-12). 40. B A: The author does not claim that they lack initiative, only that we cannot observe it in action (lines 22-29). In contrast, the chimp clearly displays its initiative as it tests the peg in various holes (lines 19-20). Finally, the theme of the first paragraph, in which the author mentions the difficulty of training orangutans, is not problem solving but the asocial and independent nature of these animals. B: Yes. In the first paragraph the author discusses Berosini’s claim that orangutans are difficult to train because they are “self-contained” (lines 4-6), that is, they care little about the “social support and approval of others” (lines 8-10). C: Limited attention span is never mentioned. D: The author does not argue that orangutans distrust humans, just that they don’t care much about receiving human (or ape) approval (lines 8-10). The fact that Berosini successfully trains orangutans [which according to the author involves a reciprocal and respectful relationship between trainer and trainee (lines 38-42)] indicates that these orangutans must trust humans to some extent.

12

41. A A: Yes. The main idea of the paragraph in which this statement appears is that training is a two-way, “symmetrical” relationship. In the subsequent paragraph, the author states that the “intelligent responsiveness of trainers” within this reciprocal relationship may be seen as respect for the animal (lines 58-61). B: The focus of the aphorism, and of the passage, is on the attentiveness of the trainer as a prerequisite for the attentiveness of the animal being trained. C: The author does refer to the natural social structure (or lack thereof) of orangutans (lines 6-7), but not in reference to training techniques. D: By “heed the animal” the author means pay attention to what the animal is saying through its behavior during training. The example given is not one of disobedience, but of “intelligent responsiveness” (lines 42-47, 54-55). 42. A A: Yes. The passage as a whole argues that we should treat and train animals with respect for their own intelligence. It is this intelligence that attracts us to animals (lines 54-55). In the cited part of the passage, the author suggests that if we come to understand that animals have something of value to say to us, we will perhaps feel less alone in the world. B: In the passage, the author proposes that we recognize the unique and varied intelligence of the apes, not that apes are just like us. Secondly, this choice is not appropriate to the main idea of the cited part of the passage. The intent of the author in those lines is not to show our similarity with apes, but to suggest that showing proper respect for other species will make us less alone in the world. C: The passage indicates just the opposite. The author wishes to eliminate sentimental attitudes toward animals (lines 6164). D: Nowhere in the passage does the author attempt to justify demanding obedience. In fact, the author suggests that we should earn, not demand the obedience or cooperation of other species by obeying or heeding what they say to us (lines 47-50). 43. D A: The author never recommends any form of punishment. This choice is inconsistent with the main idea of the passage, which stresses listening to and working together with the animal being trained. B: While this choice is not inconsistent with the overall tone of the passage, neither is it supported or suggested by any statement made by the author. C: As in B, this choice is not at odds with the positive attitude of the author towards animals, but the passage never mentions the use of rewards. D: Yes. The author describes a proper training as one that establishes a “symmetrical” (line 41) or cooperative relationship with the animal (or human) being trained (lines 38-42, 48-53). 44. B A: The author indicates just the opposite. We gain the attention of animals or humans being trained by interacting with and responding to them (lines 47-53). Therefore we should not “drop” the child or discontinue training, but instead recognize what the child is communicating through his or her lack of attention. B: Yes. The author argues that training does not end once a dog has appropriately responded to a command. Rather, it has just begun, and we must continue to attentively interact and communicate with the animal (lines 42-50). The author states that this is also true when we train other humans (line 48). C: This choice takes the statement that the trainer must “obey” or heed the trainee out of context (lines 48-50). By this the author does not mean that the trainer gives in to the demands of those being trained, but that he or she attentively listens to them in the course of continuing the training. D: Trainers may well like their trainees. However, the author does not argue that true training is accomplished through demonstrating affection, but rather through respectful attentiveness. Therefore, a good trainer should say “I hear you,” not “I like you.”

13

45. A A: Yes. In the first paragraph, the author relates the behavior of orangutans in the wild to their behavior in captivity. Their life in the wild is not conditioned by social interaction with other apes, and so they have not come to depend on the support and approval of others (lines 6-10). This attitude manifests itself in captivity as well, making it difficult to train them because they are not anxious to gain the approval of the trainer (lines 4-6). If we apply this to the new information in the question, we could reasonably conclude that wild-born orangutans do not exhibit chimp-like problem-solving behaviors in captivity because they did not need or depend on them in their natural environment. Those born in captivity would have been raised in a very different context, with different challenges, and so would exhibit different behaviors. B: This choice is incompatible with the author’s suggestion that orangutans are no less intelligent than chimpanzees (lines 29-31). C: The author does not indicate that an orangutan’s “insight oriented” approach is any less efficient than a chimp’s “problem solving” orientation (lines 31-34). D: Neither the passage nor the new information in the question gives us any evidence that the same would be true for chimpanzees. 46. D A: The passage does not suggest that orangutans cannot concentrate, just that they don’t appear to be concentrating on the task at hand (lines 22-34). They may be concentrating on other things (dreams and visions), or they may be concentrating on the task in a less visible way. Note that in the end, they do in fact successfully complete the test. Finally, since orangutans are trainable (lines 1-4, 35-36), they must be able to concentrate on the trainer’s commands. B: The comparative speed of chimps’ and orangutans’ thought processes is never an issue in the passage. The author does not tell us how long each animal takes to fit the peg in the hole, just that they accomplish it in different ways. C: The author indicates that orangutans are just as intelligent as chimpanzees (lines 29-31). D: Yes. Chimps pick up the peg and test it in different holes until they find the right one (lines 15-21). Orangutans, according to the passage, accomplish the same task through insight (lines 32-33), not experimentation.

Passage VII 47. A A: Yes. The author defines consciousness as “the facility for deep introspection that enables us to see into our own minds and those of others,” and that makes us aware of our own knowledge of the world around us (lines 14-18). Thus a person with consciousness is aware of him or her self as an individual distinct from other individuals and from the surrounding environment. An animal without self-consciousness would not have this awareness. B: Animals without consciousness have knowledge of the world around them (lines 16-17), and so may be able to learn from their experiences in that world. This choice says nothing about knowledge of self, which comes only with consciousness. C: An animal may not have language [“a vehicle for complex thought processes” (lines 23-24)], but the passage does not suggest that animals have no way to express themselves. D: This choice contradicts the author’s statement that “an animal without consciousness may ‘know’ the world it inhabits” (lines 16-17).

14

48. D A: These are two separate issues. Darwin’s theory was revolutionary because it showed us that we are “not separate from, but a part of, nature” (lines 1-7). This raised the question of why we see ourselves as unique or special, if we are in fact “nothing but an animal” (lines 7-10). Culture is one part of the answer to that question, but accepting the author’s description of the effects of culture on evolution (rapid change) does not depend on the acceptance of the claim that Darwin forced us to change our self-image through recognition of our animal nature. B: Culture could affect human evolution even if it did not contribute to our sense of specialness. The author’s argument about cultural evolution does not depend on the argument that consciousness and culture distinguish us from other animals. Even if animals had culture and consciousness, the author’s argument about the nature and speed of cultural evolution could still be valid—it would simply apply to other animals as well. C: As in choice B, the author’s claim about the effects of culture on human evolution does not depend on the acceptance of the claim that only humans have consciousness and culture [and thus curiosity about the world (paragraph 3)]. Choices A, B, and C all focus on issues from the wrong parts of the passage. D: Yes. The passage argues that cultural evolution contributes something to human evolution that genetic evolution does not: “the potential for exceedingly rapid rates of cultural change” (lines 60-63). Genetic evolution, according to the author, requires tens of thousands of years (lines 51-53). If genetic mutation did cause rapid social changes, it would undermine the author’s assertion that “what has enabled Homo sapiens to speed up the pace of change in its world is culture” (lines 53-55). 49. C A: The passage says that cultural evolution occurs through cause and effect, when a particular invention or “novel artifact” transforms entire societies (lines 68-71). Advances in art and learning are not “novel artifacts” analogous to computers (line 71), and the answer choice gives no indication that the entire society was rapidly transformed. B: This choice is wrong for the same reasons as choice A. According to the passage, cultural evolution involves “novel artifacts, which sometimes transform entire societies virtually instantaneously” (lines 68-71). No novel artifacts are involved, and we are not told that the spread of socialist ideas led to the transformation of entire societies. C: Yes. The steam engine was a new invention or novel artifact, and the ability to more easily travel and transport goods over long distances significantly transformed society (lines 68-71). D: The exploration of outer space is not currently something that dramatically affects society as a whole in the same way as long-distance travel (choice C) or the electronic computer (lines 69-71). 50. A A: Yes. The passage defines natural selection as “adaptation to shifting circumstances and environments” (lines 4041). The correct answer will not fit this definition. Here, there is no element of change or adaptation. B: This choice describes television producers responding and adapting to popular tastes. C: Here, voters adapt their electoral preferences to the changing (declining) economic climate. D: This answer describes the adaptation of language to shifts in terminology from generation to generation. 51. D Note: This is an unusual question type, a mutated version of a “weaken” question. Both the new information in the question and the correct answer choice are inconsistent with the author’s argument in the passage. However, the question does not ask for a choice that weakens the passage (A, B, C, and D all do so), but for a statement that is suggested by the information presented in the question stem. A: If most humans limit their attention to their immediate surroundings, that does not show that they gain no benefit from their capacity for consciousness. It is possible that having such a narrow focus carries some advantage. B: The issue presented in the question stem relates to humanity’s supposed “insatiable appetite for knowledge” (line 36). The relative pace of genetic and cultural evolution has no direct relevance. C: According to the passage, language allows for (not guarantees) complex thought. If most humans pay attention only to their immediate physical surrounding, that does not suggest that language does not in fact have this capacity, only that most humans do not fully utilize it. D: Yes. The passage states that civilized human beings have a thirst for knowledge about themselves and the world (lines 30-36). If the new information in the question were shown to be true, and most people cared little about anything but their immediate physical environment, it would indicate that the passage is in error, and that civilized animals (human beings) do not necessarily seek knowledge and understanding.

15

52. D A: According to the passage, language is a crucial part of culture (lines 20-24). However, the author does not suggest that culture is a prerequisite for the existence of language. Language is necessary for culture, but as far as we know culture is not necessary for language. B: This choice is only partially correct. The passage does indicate that culture is uniquely human (lines 10-13, 18-23), but not that nonhuman animals may have languages. By describing language as a “vehicle for complex thought processes” (lines 23-24), the author suggests that language too is unique to human animals. C: Language is a vehicle for complex thought (lines 23-24), but the author never implies that it is the only vehicle. D: Yes. The author describes language as a vital aspect of culture in the context of discussing how culture allows us to “participate in a collective, cumulative learning experience” (lines 22-23), and to pass on wisdom to current and future generations (lines 24-27).

Passage VIII 53. A A: Yes. The critics and proponents of democracy are arguing over what type of political regime best allows for and promotes economic development. If each side is evaluating the value of democracy based on whether or not it leads to economic growth (lines 13-16, 21-23, 36-38), each position must be based on an assumption that growth is a necessity, especially for developing nations (lines 16-19, 32-34). B: The first set of scholars described in the passage believes that democracy and economic growth are incompatible (lines 1-3). C: The critics of democracy do not see social benefits as necessary; they propose holding down wages to force a reduction in consumption (lines 6-10). These scholars claim that democratically-elected leaders who implemented such policies would be removed from office by consumers who do not perceive limited consumption to be a social benefit (lines 712). D: The critics described in the second half of the passage argue that democracy, not authoritarianism, best promotes necessary growth (lines 55-63). 54. B A: The opinion cited in the question is challenged by the proponents of democracy in lines 41-54, but with no reference to an increase in production. B: Yes. In the fifth paragraph, the advocates of democracy argue that consumption in the areas of health and education also constitutes investment in human capital (lines 48-51). Thus, some forms of consumption contribute both to investment surplus and human welfare. C: The critics of the opinion expressed in the question argue that consumption and investment are not mutually exclusive (lines 46-54). They do not, however, make any claims about allowing for steady consumption. D: The critics of the opinion cited in the question make no reference to decreased demand (see paragraph 5). 55. A A: Yes. The premises stated in the question indicate that the governments presiding over recent successful development have all intervened in the economy, and that development is most likely to occur in the presence of political stability. Therefore, it is reasonable to conclude that any recent case of successful development will include these two conditions. B: This choice is too extreme. The premises reproduced in the question state that development is best promoted in conditions of political stability, but not that stability is absolutely necessary for development. C: The premises draw no direct connection between stability and intervention. The scenario of an unstable interventionist government is not inconsistent with the position described in the question. D: See the explanation for choice C.

16

56. D A: The world market has no direct relevance to this history. B: We do not know from the story presented that China in the 1950s was politically unstable. C: The new information in the question gives no evidence of a lack of investment. D: Yes. The first view presented in the passage asserts that intensive state intervention in the economy comes first in the course of economic development, while citizen participation can only come later (lines 36-40). The history provided in the question indicates that the two can coexist; the state redistributed land and organized the collectives, and yet those collectives were democratic in nature. 57. A A: Yes. The claim that democracy hampers economic growth is predicated in part on the belief that growth requires an investable surplus, and that the only way to increase this surplus is to limit consumption (lines 3-7). The percentage of wages kept in savings accounts would not be available for consumption (spending), and so would be available for investment (lines 5-6). Note that these are low-income workers; it is likely that the required percentage exceeds their current rate of savings. B: The passage indicates just the opposite. The scholars who argue that democracy limits growth claim that both a reduction in consumption (which would likely result when low-income people are forced to increase their savings) and political stability are necessary conditions for growth. Thus, it would be logically inconsistent for these scholars to claim that policies limiting consumption lead to instability. C: The passage indicates just the opposite—see the explanation for choice A. D: This choice misrepresents the relationship between consumption and democracy presented in the first half of the passage. The scholars who argue that democracy is incompatible with economic growth claim that a necessary longterm reduction in consumption cannot occur in a democratic state, because the politicians who impose it will be voted out of office (lines 7-12). This does not mean that forced limitations on consumption prevent democracy from developing. 58. B A: According to this argument, an authoritarian government does not need popular support (lines 7-12). B: Yes. Those who believe democracy cannot coexist with economic growth base this belief in part on a claim that democracy opens up a regime to pressures from different groups in society, making it difficult to maintain the stability necessary for development (lines 13-20). An authoritarian regime would be less vulnerable to this kind of public pressure. C: As described in the passage, those who believe democracy holds back economic growth are more concerned with the economic development of the nation as a whole than with the concerns of individual groups within the nation (lines 1619). The author never discusses what the views of these scholars might be regarding the protection of minority groups. D: The author does not explicitly address this issue in the first half of the passage. However, the scholars who believe democracy hurts growth, as described here, do not appear to be concerned with the issue of fairness.

Passage IX 59. A A: Yes. In lines 16-20, the author states that both primal spirits (Angra Mainyu and Ahura Mazda) deliberately chose between good and evil, and that all persons are required to make the same choice. B: Only Ahura Mazda is eternal (lines 10-12); the evil Angra Mainyu will be destroyed in the great battle between good and evil at the end of time (lines 21-23). C: Angra Mainyu is “equally uncreated” (lines 14-15), not a creation of Ahura Mazda. D: This is the belief of the Zurvanite heretics (lines 43-46), not of all Zoroastrians. 60. B A, C, and D: The author draws no comparison between Indian and Babylonian conceptions of time (Indian conceptions are not mentioned in the passage) and does not claim that the Zurvanites were influenced only by Babylonian ideas. Thus this discovery would have no effect on the author’s argument, as stated in choice B. 17

61. B A: According to the author, Zoroaster’s teaching was unprecedented because he proclaimed Ahura Mazda as the “one uncreated god, existing eternally” (lines 9-12). The author discusses the utter separateness of the primal spirits later in the passage in the discussion of the Zurvanite heresy (lines 52-56). B: Yes. The Zurvanite’s believed that Ahura Mazda and Angra Mainyu were twins from the same father, thus violating Zoroaster’s tenet that the two were completely “distinct by origin and nature” (lines 52-56). This is the basis of the author’s accusation of heresy. C: This is a different issue. The author does not give any explicit support for the claim that the Zurvanites were familiar with the Babylonian idea of time; it is simply asserted as a fact (lines 29-33). D: Again, this is a separate issue. The author discusses Zoroaster’s fundamental doctrine of separateness to demonstrate the heretical nature of the Zurvanites in lines 52-56. With the word “moreover” in line 56, the author moves on to a new issue, free will. 62. D A: This choice is too narrow to serve as the main point. The author mentions Zoroaster’s vision as part of the introductory description of how the religion came into being (lines 3-7). B: This choice is also too limited. The origin and nature of Zoroastrian religion are described in order to set the stage for the author’s discussion of the Zurvanite heresy. C: This is the view of the Zurvanite heretics (lines 37-40); it does not characterize Zoroastrianism as a whole. D: Yes. The author describes the teachings of Zoroaster in the first two and a half paragraphs. The rest of the passage explains how the Zurvanites heretically broke with those teachings. 63. C A: The author does not claim that Zoroaster was the first to proclaim the end of history, just that this claim was one of the “most striking elements” in his teaching (lines 27-29). This discovery would not be inconsistent with the passage. B: The author suggests that the Zurvanite conception of time as a sentient being was based in Babylonian speculations that time is cyclical, and so unending (lines 29-35). An early Zurvanite denial that history has an end (i.e., a claim that history and time do not end) would strengthen, not weaken the author’s argument. C: Yes. The author very strongly argues that the Zurvanites committed “a deep and grievous heresy” by claiming that the two primal beings were brothers (lines 52-56). If Zoroaster himself made this claim, the Zurvanites would not in fact have “betrayed Zoroaster’s fundamental doctrine.” D: The author does not argue that Zoroaster had no premonitory inkling of what heresies were to come. This discovery would have no effect on the author’s position. 64. D A: The passage tells us nothing about all religions, just this one religion. B: We know that Zoroastrianism did not maintain wholly consistent doctrines, as it gave rise to the Zurvanite heresy. The passage gives no clue as to whether or not other religions did so. C: There is no discussion in the passage of justification of any, and certainly not of all, religious doctrines. D: Yes. The Zurvanites (who were converts to Zoroastrianism, lines 25-27) interpreted Zoroaster’s writings in a way that was inconsistent with Zoroaster’s own beliefs (lines 40-43). 65. C A: The Zurvanites and the Orphic Greeks both existed in the 6th century B.C. (lines 26-27). Therefore, one group could have influenced the other. B: See the explanation for choice A. C: Yes. Zoroaster lived between 1400 and 1200 B.C., roughly 6 to 8 centuries before the existence of the Orphic Greeks. Thus, it is impossible that the Orphic Greeks had an influence on Zoroaster. D: See the explanation for choice C. Zoroaster predates the Orphic Greeks. Therefore, it is not inconceivable that he had some influence on them.

18

PHYSICAL SCIENCES Passage I 66. D. As the mass on the hook falls, its gravitational potential energy decreases and is converted to kinetic energy, both of the falling mass and of the wooden block on the table. 67. A. According to the data in Table 1, whether the base area is 0.001 m3, 0.002 m3, or 0.003 m3, the threshold mass for the wood block is essentially the same: 0.049 kg. Whether the base area is 0.001 m3, 0.002 m3, or 0.003 m3, the threshold mass for the stone block is the same: 0.068 kg. And whether the base area is 0.001 m3, 0.002 m3, or 0.003 m3, the threshold mass for the steel block is the same: 0.055 kg. We can conclude that the threshold mass (and, therefore, static friction) depends on the type of material the block on the table is composed of, but not on its base area. 68. C. The gravitational force on the sliding block does zero work since it’s perpendicular to the displacement of the sliding block. However, the tension in the string does positive work, and the force of kinetic friction does negative work. 69. B. First, we can eliminate choice A. The experiment described in the passage does not have metal surfaces in contact; part of the experiment has a steel block sliding on a wooden board. Since the threshold mass was smallest for the wood block, we conclude that the attractive molecular forces between surfaces were weakest between wood surfaces. 70. A. If the wooden board on which the blocks slide is coated with lubricant, the static friction force should be smaller with any block material. Therefore, we’d expect the threshold mass would decrease in all cases. 71. D. The force of kinetic friction on the sliding block has magnitude Ff = µmg, where m is the mass of the block. This force remains constant while the block slides. The net force on the sliding block is FT – Ff, where FT is the tension in the string. Since FT remains constant (as we’re told in the question), the net force on the block is also constant. Therefore, the acceleration of the sliding block, a = Fnet/m, is constant, which means its speed will increase linearly with time (v = at). 72. D. The experiment described in the passage measures the minimum mass that must be placed on the hook to get the block to begin sliding. This procedure allows the researchers to deduce the maximum force of static friction between the various blocks and the wooden board, since static friction attains its maximum strength just before the block starts to slide. To do the same thing by tipping the board, the researchers will want to measure the angle at which the block starts to slide down. If this angle is θ, then the force of static friction just before the block begins to slide has magnitude µsmg cos θ, and the gravitational force on the block has magnitude mg sin θ. Just before the block slides, these forces balance, so µsmg cos θ = mg sin θ, which means µs = (sin θ)/(cos θ) = tan θ. So, by measuring the angle that the board makes with the horizontal, the researchers can determine the coefficient of static friction: µs = tan θ.

Passage II 73. A. The lone pair of electrons on the nitrogen atom in NH3 is donated to the transition metal cation to form a coordinate covalent bond, so the answer is A. (While choices B, C, and D are all true statements, they don’t answer the question.) 74. C. A compound’s boiling point depends on the strength of its intermolecular forces and the ability of its molecules to enter the gaseous state from the liquid state. Choice A is eliminated since the reactivity of a compound with other compounds is irrelevant; we’re interested here only in the behavior of the pure compound. Choice B is eliminated since, for one thing, electronegativity is reserved for an individual atom (not a molecule) and its propensity for attracting electrons when forming a bond with another atom. But once again, the concept of electronegativity does not apply here, since a compound’s boiling point does not depend on its ability to form bonds with atoms of another compound. And even though choice D is a true statement, it does not answer the question: the N2 and O2 molecules will still contain a triple bond and double bond, respectively, in the gaseous phase.

19

75. A. First, eliminate choice D; adding a catalyst to a reaction that’s already at equilibrium will have no effect. By Le Châtelier’s principle, the addition of a reactant will shift the equilibrium to the right; so, if H2 is added, more NH3 should be formed. Choice B is wrong since a decrease in pressure will cause the reaction to favor the side with the greater number of gas molecules; in this case, the equilibrium would be shifted to the left, consuming NH3. Choice C is wrong since Reaction IV is exothermic (∆H is negative). We can therefore think of heat as a product, so increasing the temperature will cause the equilibrium to shift to the left, consuming NH3. 76. D. Fractional distillation separates the components of a mixture by their boiling points. The most volatile compound— the one with the lowest boiling point—will be isolated first. 77. B. According to Reaction I, one mole of O2 molecules reacts with one mole of sulfur atoms to form one mole of SO2. Since the reaction begins with 32 g of O2 (which is 1 mole of O2), the reaction could consume one mole of S atoms. Sulfur has a molecular weight of just 32.1 g/mol, so having 36 g of S means that there will be some unreacted sulfur. Since all the O2 has already been consumed, it is the limiting reagent.

Passage III 78. D. The key to answering this Doppler Effect question is to find a situation in which the transmitter and receiver have the same relative velocity. The original situation given in the question has the transmitter and receiver moving away from each other at 300 m/s, so our task is to find another situation in which the transmitter and receiver are also separating at a relative velocity of 300 m/s. This immediately rules out choice C. We can also rule out choices A and B: If the receiver is moving in the opposite direction as the jet, then their relative velocity of separation will be greater than the velocity of the jet itself, which is 600 m/s. The answer must be D: If the receiver moves at 300 m/s toward the jet, then the jet is moving away from the receiver at a relative velocity of only 600 – 300 = 300 m/s. 79. C. According to the data in Table 1, the magnitude of the “Change in f ” increases as f increases. Since a frequency, f, of 2.5 × 106 Hz is between 2.00 × 106 Hz and 3.00 × 106 Hz, we’d expect the change in frequency to be between –1.79 Hz and –2.68 Hz (the values for f = 2.00 × 106 Hz and 3.00 × 106 Hz, respectively). Only choice C, –2.2 Hz, falls in this range. 80. A. Look at Table 2. In each case, f and λ were constant; only the speed with which the transmitter moves away is varied. We see that as the speed increases, so does the change in λ (and vice versa). Therefore, if we graph speed vs. “change in λ,” the graph should increase. Of the choices given only the graph in choice A increases. 81. C. Look again at Table 2: As the speed increases, so does the change in λ. If the change in λ increases, then the wavelength received also increases, so, by definition, the distance between adjacent peaks increases. 82. A. First, eliminate choices B and C. Sound waves have frequencies of less than 20,000 Hz, while the radio waves used in these experiments have frequencies in the millions of hertz. Therefore, B is false (and, therefore, so is C). There is nothing in the passage to support the statement in D, so the best response is A. It is a true statement (since radio waves travel at the speed of light), and the relative velocities of the transmitters and receivers in these experiments are hundreds of meters per second, a significant fraction of the speed of sound but only a miniscule fraction of the speed of radio waves. 83. A. If the transmitting and receiving jets are flying side by side, then the velocity of either one is zero with respect to the other. If there is no relative velocity, there is no Doppler shift. 84. D. Since the wavelength received is shorter than the wavelength emitted, the frequency received is higher than the frequency emitted. We always associate a frequency shift upward with relative motion toward (whether it’s with sound waves, radio waves, or light waves). Of the choices given, only choice D could explain relative motion toward.

20

Passage IV 85. D. The “millivolt recorder” simply measures the voltage drop across the resistor R, which is proportional to the current I through it (V = IR), which is in turn proportional to the number of electrons that flow through it [I = Q/t = (ne)/t]. If another gas were in the stream that also undergoes a reduction similar to that of oxygen gas, then more electrons would be produced (as a result of Equation 3 which follows from a reaction similar to Equation 1), thus increasing the current in, and the voltage drop across, the resistor R. Therefore, the voltage drop would be greater than what it would be if only oxygen were present. As a result, the researcher would think that the oxygen concentration is greater than it actually is (since “the oxygen concentration is proportional to the potential”), thus decreasing the accuracy of the oxygen determination. 86. C. Use the Combined Gas law: P1V1/T1 = P2V2/T2. At T1 = 30°C = 303 K and P1 = 756 mmHg, we have V1 = 10 L. So, at STP (P2 = 760 mmHg and T2 = 0°C = 273 K), the volume of the gas is V2 = V1 ⋅

P1 T2 756 mmHg 273 K (10)(756)(273) ⋅ = (10 L) ⋅ ⋅ = L P2 T1 760 mmHg 303 K (760)(303)

87. B. According to the passage, reduction occurs at the silver electrode (immediately eliminating choice D), and “the electrochemical reaction is completed at the cadmium electrode.” Therefore, the cadmium electrode must be the site of oxidation, so it is the anode. 88. C. One percent is equal to 1/100. Multiplying both the numerator and denominator of this fraction by 104 = 10,000, we see that 1% = 104/106 = 10,000 parts per million. 89. D. Oxygen is reduced in Equation 1, its oxidation state decreasing from 0 (in O2) to –2 (in OH–). The carbon atom in CH4, however, is already in a –4 (its lowest) oxidation state and won’t be reduced further in a reaction with water. Therefore, OH– ions would not be produced, so Equation 3 would not occur, and no current would flow through R for the voltmeter to register. Note: Choice C is false since hydrogen is in a +1 oxidation state in CH4 and thus not in its lowest oxidation state.

Independent Questions 90. A. The combination of (stoichiometric amounts of) a strong acid with a weak base will leave the aqueous solution the most acidic and thus with the lowest pH. 91. C. The pressure at depth h below the surface of a liquid is P = P0 +ρgh, where P0 is the pressure on the surface (which is Patm if the container is open to the atmosphere) and ρ is the density of the liquid. Since P = 250 Pa when h = 5 cm, we have 250 Pa = P0 + ρg(5 cm); and since P = 450 Pa when h = 10 cm, we have 450 Pa = P0 + ρg(10 cm). If we multiply both sides of the first equation by 2, we get 500 Pa = 2P0 + ρg(10 cm), and then subtracting the second equation from this one, we get 50 Pa = P0. Now, substituting this result into the first equation gives 250 Pa = 50 Pa + ρg(5 cm), so ρg = 200 Pa/5 cm = 40 Pa/cm. Therefore, P = (50 Pa) + (40 Pa/cm)(h). [You can check that this formula works for all the entries in the table given with the question.] If we now have a liquid with twice the density of the first, then ρ′g = (2ρ)g = 2(ρg) = 2(40 Pa/cm) = 80 Pa/cm, so the pressure at depth h below the surface of the second liquid is given by the formula P = (50 Pa) + (80 Pa/cm)(h). When h = 10 cm, this formula gives P = (50 Pa) + (80 Pa/cm)(10 cm) = 850 Pa. 92. C. Let V be the volume of the object. Since the object “suffers an apparent loss of 5 grams” when immersed, the object experiences an apparent loss of weight of (5 grams)(g), so the buoyant force on the object must have magnitude (5 grams)(g). The buoyant force on the object is given by the expression ρfluidVg (note that Vsub = V since the object is immersed; that is, totally submerged). Therefore, (5 grams)(g) = ρfluidVg, so ρfluidV = 5 grams. The mass of the object is ρobjectV, which we’re told is 15 grams, so ρobjectV = 15 grams. We can now combine these equations to find that

ρobject V ρ fluid V

=

15 grams 5 grams



ρobject ρ fluid

= 3 ⇒ ρ object = 3ρ fluid = 3ρ benzene = 3(0.7ρ H2O ) = 2.1ρ H2O

21

93. A. Since the metal is at its exact melting point, it is ready to undergo a phase transition from solid to liquid. Since the temperature of a compound remains constant during a phase change, choices C and D are eliminated. A small amount of additional heat energy absorbed by the sample will thus melt some of it, but the temperature will not change. It is safe to assume that if the 0.1 kg sample of metal is held over the flame for only a “fraction of a second,” there won’t be enough heat energy to melt all of it, so A is a better response than B.

Passage V 94. C. The passage gives the molar mass of KHP as M = 204.2 g/mol, and Table 2 shows that Student A had 0.55 g of KHP. Therefore, the number of moles of KHP present in the sample of KHP used by Student A is n=

m 0.55 g 0.5 g 1 1 1 = ≈ = g mol = × g mol = 0.25 × 10 −2 g mol = 2.5 × 10 −3 g mol M 204.2 g mol 200 g mol 400 4 100

95. C. Since the temperature rose during the mixing process, ∆H must be negative for Equation 1, so Item I is true. This eliminates choices B and D. Since an aqueous solution of a dissolved solute has greater disorder than the separate pure solid NaOH(s) and pure liquid H2O(l), the entropy increased during the mixing process, so Item III is also true. Therefore, the answer must be C. [Since the solid NaOH dissolved spontaneously in solution, the value of ∆G must be negative for this reaction, which is why Item II is false.] 96. A. The passage states that the instructor “prepared a solution of NaOH(aq) by dissolving 8 g of NaOH(s) (MM = 40.00) in 2 L of H2O.” Eight grams of NaOH(s) thus represents 8/40 = 1/5 = 0.2 mol; dividing this by 2 L, we find that [NaOH] = (0.2 mol)/(2 L) = 0.1 mol/L = 0.1 M. 97. D. At the equivalence point of the titration of a weak acid by a strong base, the solution is basic (that is, pH > 7). 98. D. To find the conjugate base of chlorobenzoic acid, HC7H4ClO2, simply remove an H+; the result is C7H4ClO2–. 99. B. First, eliminate choices C and D: According to the formula given in Table 1, succinic acid is diprotic, not triprotic. And, since the acid is diprotic, more base will be required to neutralize this acid than if the acid were only monoprotic, so the answer must be B, not A. 100. A. If the KHP sample contains water, then the mass of the sample will be greater than the mass of the KHP alone. As a result, when the student uses this to calculate the number of moles of KHP in the sample, the student will derive a value that’s greater than the true value. Using Equation 2, the student would then conclude that more NaOH is needed to reach the equivalence point (since the student derived a greater-than-actual value for the number of moles of KHP). Therefore, the molarity determined for the NaOH(aq) solution would be too high, because the number of moles of KHP used in the calculation is greater than the actual number of moles of KHP titrated. This statement is equivalent to choice A.

Passage VI 101. C. Nothing in the passage supports choice A or B. Choice D must be eliminated, since momentum is conserved in the collision and, in addition, momentum is not converted to energy. Choice C says it best: The kinetic energy of the gas from the ordinary companion star that collides completely inelastically with the surface of the white dwarf becomes heat energy. 102. A. By equating mg with GMm/r2, we see that the gravitational field strength, g, at the surface of a star is given by the formula g = GM/r2, where M is the mass of the star and r is its radius. The passage states that a white dwarf has the same mass, M, as the Sun, but a much smaller radius, r. Since M is the same but r is much smaller for a white dwarf, the value of g = GM/r2 is much greater at the surface of a white dwarf than at the surface of the Sun. Thus, choice A is the answer. Choice B is false, since all three hypotheses include the fact that the white dwarf does experience nuclear fusion, and nothing in the passage supports choice C or D.

22

103. B. First, eliminate choices A and C: Fusion of bare nuclei is not the same as bonding of atoms containing electrons. And choice D is false, since hydrogen nuclei (which are simply protons) are much lighter than carbon nuclei, and therefore should move much faster, on average, than carbon nuclei at the same temperature. The answer must be B: It is easier to fuse two hydrogen nuclei (two protons) than two carbon nuclei, because each hydrogen nucleus is only a single proton, while each carbon nucleus contains six protons. The electrical repulsion is much lower between a pair of single protons than between a pair of 6-proton nuclei. 104. D. The following balanced nuclear reactions show that decay of either 56Ni or 56Co is accompanied by the emission of 56 0 + 56 0 + (that is, both 56Ni and 56Co undergo β+ decay). a positron, e+: 56 and 56 28 Ni → 27 Co + +1 e 27 Co → 26 Fe + +1 e 105. D. While choices B and C are both true, they don’t answer the question. As for choice A, the second sentence in the paragraph describing Hypothesis III would, if anything, cast doubt on this statement. The best response is D: The “runaway” (that is, very rapid) nature of the fusion reaction would be manifested by a sudden, large increase in brightness.

Passage VII 106. B. The oxidation of Zn(s) has a potential of +0.76 V (reverse the reaction given in Table 2). Combining this with the reduction of Cu2+ (E° = +0.34 V) gives a cell voltage of +1.10 V, which is the measurement made by Student 3: oxidation: reduction:

Zn(s) → Zn2+(aq) + 2e– 2+ – Cu (aq) + 2e → Cu(s)

E° = +0.76 V E° = +0.34 V

Cu2+(aq) + Zn(s) → Cu(s) + Zn2+(aq)

E° = +1.10 V

107. D. Since the molar mass of the white solid used by Student 4 is 58.5 g/mol, a mass of 29.25 g represents 0.5 mol of the solid. If this is present in an aqueous solution of volume 250 mL = 0.25 L, then the concentration is (0.5 mol)/(0.25 L) = 2 M. 108. C. Consider each molecule of Fe2(SO4)3 as composed of 2 Fe3+ cations and 3 SO42– anions. Thus, iron is in a +3 oxidation state—that is, iron(III)—eliminating choices A and B. Since SO42– is sulfate (SO32– is sulfite), the compound is iron(III) sulfate. 109. B. Since the reaction is spontaneous, the cell voltage must be positive (Ecell > 0 ⇒ ∆G < 0 ⇒ spontaneous). Therefore, the half-reactions must be reduction: oxidation:

Ag+(aq) + e– → Ag(s) Al(s) → Al3+(aq) + 3e–

E° = +0.80 V E° = +1.66 V

Balancing the electrons in these reactions, we multiply the reduction half-reaction by 3 (but not its potential!): reduction: oxidation:

3 Ag+(aq) + 3e– → 3 Ag(s) Al(s) → Al3+(aq) + 3e–

E° = +0.80 V E° = +1.66 V

Giving the overall reaction, 3 Ag+(aq) + Al(s) → 3 Ag(s) + Al3+(aq)

E° = +2.46 V

Since pure solids are omitted from the equilibrium and reaction-quotient expressions, we have Q=

[Al 3+ ] [Ag + ]3

23

Passage VIII 110. B. The molar mass of KNO3 is 101.g/mol and that of H2O is 18 g/mol. Therefore, in a 1 L solution whose KNO3 concentration is 226.5 g/L and whose H2O concentration is 906.1 g/L (see Table 1), the mole fraction of KNO3 in the solution is XKNO 3 =

nKNO 3 nKNO 3 + nH 2 O

=

(

(

)(1 L) )(1 L) )(1 L) + (

226.5 g L 101.1 g mol

226.5 g L 101.1 g mol

906.1 g L 18 g mol

111. C. Each of the compounds in choices A, B, and D contains a common ion with KNO3—that is, either K+ or NO3–. By the common-ion effect, any of these added to a saturated solution of KNO3 would cause precipitation of crystals. However, Choice C, NH4Cl, has no common ion with KNO3 and would not cause precipitation. 112. D. The passage states that the condosity of a solution is the molar concentration of sodium chloride that has the same conductance as the solution. According to the data in Table 1, the student’s KNO3(aq) 2.241 M solution has a condosity of 2.49 M. So, if we plot conductance vs. molarity, the fact that the student’s solution and the 2.49 M NaCl(aq) solution have the same conductance means that they’d lie on the same horizontal line in the plot, eliminating Points A and C. And since the molarity of the student’s solution is less than that of the NaCl(aq) solution, the KNO3(aq) point should be to the left of the NaCl(aq) point in the plot. Thus, the answer must be Point D. 113. C. According to Table 1, the solute concentration of the KNO3(aq) solution is 226.5 g/L, which is equivalent to 22.65 g in 0.1 L (= 100 mL). 114. D. Since the concentration of the KNO3(aq) solution is 2.241 M, the student’s 1-liter solution contains 2.241 moles of KNO3. Since each mole of KNO3 contains 6.02 × 1023 K+ ions, 2.241 moles of KNO3 contain (2.241) × (6.02 × 1023) ≈ 1.3 × 1024 K+ ions.

Independent Questions 115. C. The balanced reaction is CH4(g) + 2 O2(g) → CO2(g) + 2 H2O(g). Therefore, the mole fraction of H2O(g) in the product mixture is nH 2 O 2 2 XH 2 O = = = nCO 2 + nH 2 O 1 + 2 3 Applying Dalton’s law, we find that the partial pressure of H2O(g) in the product mixture is pH 2 O = XH 2 O ⋅ ptotal = 23 ⋅ (1.2 torr) = 0.8 torr

116. A. Since carbon, hydrogen, and oxygen have molar masses of 12 g, 1 g, and 16 g, respectively, the sample described in the question contains 1 mole of C atoms, 2 moles of H atoms, and 1 mole of O atoms. Therefore, the formula for this compound must be CH2O. (Note that choices C and D can be eliminated immediately since they’re not empirical formulas. An empirical formula contains the smallest whole number subscripts that have the same ratio as the subscripts on the atoms in the compound’s actual molecular formula.) 117. D. If the pipe is open at both ends, then both ends are displacement antinodes (A). The simplest standing wave—the fundamental standing wave—must therefore have a node (N) in the center of the pipe: A N A. The distance from one antinode to the adjacent antinode on a standing wave is always equal to one-half the wavelength. So, if the length of the pipe is 1 m, the distance from one antinode to the other antinode on the fundamental standing wave is 1 m. Since this must be equal to λ/2, we find that λ = 2 m.

24

118. A. When the voltage is turned on, charge begins to build up on the capacitor plates. The rate at which the charge builds is not steady, but gradually decreases, since it’s more difficult to place additional charge on a capacitor that already has some charge, because we’re “fighting against” the electric field in the capacitor created by the charge that’s already there. Then, once the voltage is turned off, the charge gradually leaks off, quickly at first and then more slowly near the end. Since the voltage between the capacitor plates is proportional to the charge (Q = CV), the voltage between Points A and B will exhibit the same behavior as the charge on the capacitor. Thus, the voltage rises, at a rate that decreases with time, then the voltage drops, also with a rate that decreases with time. This is shown in graph A. 119. B. Mechanical waves transport momentum and energy from one point to another, but they do not cause particles of the medium to travel great distances. For example, the wave in a rope does not transfer rope particles from one end of the rope to the other, and water waves created thousands of miles offshore do not transport water that distance.

Passage IX 120. A. The total momentum of the colliding objects is conserved in a collision. It makes no sense to say that impulse (choice D) was conserved, since impulse is force multiplied by time; there was no contact force between the continents before their collision, so there was no impulse before the collision. Also, we know that energy was not conserved; only in elastic collisions, where the objects simply bounce of each other, is kinetic energy conserved. But these continents did not cleanly bounce of each other: The formation of the mountain ranges in North America and Africa tells us that. As for potential energy (gravitational, supposedly), there is no reason to believe (or evidence to support) the extraordinary contention that the total potential energy of the continents before the collision was equal to their total potential energy afterwards. Collisions conserve the momentum of the system; energy (in either form) is not necessarily conserved. 121. C. First, we can eliminate choice A. The second paragraph of the passage tells us that 40K is radioactive, so no organic material need be present to use this type of radioactive dating procedure to ascertain the age of the rock. [Radiocarbon (carbon-14) dating, on the other hand, does require the presence of organic material. However, the half-life of carbon-14 is only about 5700 years, so radiocarbon dating is not a reliable method for determining the age of specimens (such as rocks) that are hundreds of millions of years old. Radiocarbon dating is used only for artifacts that are at most tens of thousands of years old.] The second paragraph contains the clue to answering this question. We’re told that “the age of the rock can be determined by measuring the amount of radioactive potassium in the rock and the amount of its decay product, argon gas in the sample.” Clearly then, it is measuring the relative amount of parent and daughter nuclei that is critical. So, to obtain an accurate dating, researchers would want the decay product to be trapped in the rock since its formation, so all of it can be measured to determine how much decay has occurred. Choice B is eliminated because it says “any gases” (only the relative amounts of potassium and argon gas are necessary for this procedure), and nothing in the passage supports choice D. 122. A. If the continents stick together after the collision, then by Conservation of Momentum, we have m1v1 + m2v2 = (m1 + m2)v′, so v′ = (m1v1 + m2v2)/(m1 + m2). 123. D. The passage states that the daughter nucleus of the decay of 40K is 40Ar. The following balanced nuclear reaction 40 0 + shows that the decay of 40K is accompanied by the emission of a positron, e+: 40 19 K → 18 Ar + +1 e . 124. B. By Newton’s Third law, the force that the first continent exerts on the second continent is equal in magnitude to the force that the second continent exerts on the first. (The masses of the continents are relevant to their resulting accelerations, but not to the magnitude of the force the continents exert on each other. Newton’s Third law says nothing about mass.)

25

Passage X 125. A. First, eliminate choices C and D. Changing R will not affect the voltage between the electrodes. As the passage states, “the potential difference between the cathode and anode is approximately equal to the battery voltage, V = 50 V.” However, the electric field between the electrodes does depend on L, the distance between the plates; in fact, we’re told that E = V/L. So, to decrease E, we should increase L. 126. B. Since the electron is ejected by a photon whose energy is only slightly greater than the work function of the metal, there is very little energy left over as kinetic energy of the electron. So, all of the photoelectron’s energy after its release from the cathode is its electrical potential energy. This energy is then converted to kinetic energy as the electron accelerates across the gap to the anode. 127. A. Each photon with energy above the metal’s work function that is absorbed by the metal surface will eject an electron. So, if more such photons strike the cathode, more electrons will be ejected. 128. C. Since the cathode is negatively charged and the anode is positively charged, the electric field between the plates points to the left in Figure 1. The force on the electron is FE = qE = eE, directed to the right. Because the electron feels a force to the right, it will accelerate to the right, moving across the gap toward the anode. 129. B. The power dissipated by a resistor, R, that carries a current I is given by the equation P = I2R. Since R = 100 Ω, the power dissipated when I = 10–3 A is P = (10–3 A)2(100 Ω) = (10–6 × 102) W = 10–4 W. 130. C. The net movement of charge through an element in a circuit—which occurs here when an electron moves from the cathode to the anode—is the definition of current. 131. D. Increasing the frequency of each incident photon increases the energy of each incident photon. However, each absorbed photon can cause the ejection of only a single electron, so increasing the energy of each photon will not change the number of electrons ejected (only changing the number of incident photons would do that). This eliminates choices A and B. If φ denotes the cathode metal’s work function (which is a constant for each metal), then the maximum kinetic energy of each ejected electron is equal to Ephoton – φ. So, if Ephoton is increased, then so will KEmax for the ejected electrons. If the ejected electrons have more kinetic energy, they’ll have a greater speed.

Passage XI 132. B. According to the data in Table 1, the element X has a melting point of 839°C and a boiling point of 1484°C. Therefore, at the intermediate temperature of 1200°C, X must be melted and thus a liquid. 133. A. Molarity is equal to moles of solute divided by liters of solution. If we know the density of the saturated solution in, say, grams per liter of solution, then if we divide this by the known molar mass of the solute (in grams per mole of solute), we’ll get: (grams/L) ÷ (grams/mol solute) = mol solute/L, which is the molarity. 134. B. It is reasonable to conclude that the oxide of X under study is a neutral molecule, so choices A and D may be eliminated. Since oxygen has a much greater electronegativity than X (according to Table 2), oxygen should have a much greater electron density than X in the oxide molecule. This is also consistent with the statement in the passage that the oxide is an ionic compound. Therefore, we expect X to have a positive charge, and oxygen a negative charge in the oxide. 135. C. The passage states that the evolved gas is flammable, so the answer must be C (hydrogen gas). [Note: Oxygen is not flammable since it does not reaction with oxygen to produce heat.] 136. B. Since the difference between the second ionization energy and the third is so large, it’s reasonable to conclude that element X has two valence electrons. The third ionization energy is so high because an electron is being removed from a +2 cation with a noble-gas configuration. Therefore, X is a Group 2 element; that is, it is an alkaline earth metal.

26

Independent Questions 137. C. Choice A is eliminated since it’s too general: An electron may change its orbit by moving into a higher-energy orbit, but this process would correspond to an absorption, not radiation of energy. Choice B is also eliminated: An object (such as an electron) in a circular orbit is always undergoing acceleration, but in the Bohr model, this type of charge acceleration does not cause energy to be radiated. And choice D is wrong, since to move to an orbit of larger radius, an electron must absorb energy. The answer is C. 138. A. “Fusion” means melting, and the heat of fusion is the amount of heat absorbed when a substance is transformed from the solid to the liquid phase. 139. B. If the current through the 2 Ω resistor is 2 A, then the voltage drop across this 2 Ω resistor is V = IR = (2 A) × (2 Ω) = 4 V. Since the 4 Ω resistor is in parallel with the 2 Ω resistor, the voltage drop across the 4 Ω resistor must also be 4 V. Therefore, the current through the 4 Ω resistor is I = V/R = (4 V)/(4 Ω) = 1 A. This means that the total amount of current in the parallel combination is 2 A + 1 A = 3 A, so this must have been the current in the 3 Ω resistor. 140. B. According to Bernoulli’s Equation, fluid pressure in a horizontal pipe is highest where the flow speed, v, is lowest. To find the point of minimum flow speed, we use the Continuity Equation, which says that Av is constant, where A is the cross-sectional area of the pipe. The speed v is lowest at the point where the cross-sectional area is greatest, and this would occur at the point where the pipe diameter is greatest. 141. B. For a plane mirror, the image distance behind the mirror, i, is equal to the object distance in front of the mirror, o (disregarding the sign convention that merely tells us that the image formed by a plane mirror is behind the mirror and is virtual). Therefore, the total distance from the person to his or her image is o + i = o + o = 2o. The question tells us that this distance, 2o, must be at least 300 cm, so o must be at least 150 cm. 142. D. The pH of a solution is equal to –log [H3O+]. If this is equal to 6, then log[H3O+] = –6, so [H3O+] = 10–6 M.

27

BIOLOGICAL SCIENCES Passage I 143. A. The question states that morphine increases parasympathetic impulse traffic to the iris, and this, as indicated in the last paragraph of the passage, would cause constriction of the pupil (choices C and D can be eliminated). Furthermore, the passage states that acetylcholine is the neurotransmitter released by the parasympathetic system at the organ level (choice B can be eliminated). 144. A. Since atropine is preventing acetylcholine (the normal parasympathetic neurotransmitter) from binding to its receptor, it is blocking the effects of the parasympathetic system, and this is described in the question as a passive mechanism (choices B, C, and D can be eliminated). 145. D. Acetylcholine is the parasympathetic neurotransmitter, thus if it is removed from circulation faster than norepinephrine (the sympathetic neurotransmitter), parasympathetic effects must turn off faster than sympathetic effects. Choices A, B, and C all describe sympathetic effects; only choice D, stimulation of digestive secretion, is a parasympathetic effect, and would be inactivated the fastest. 146. C. Any nerve fiber that causes an effect on an organ is a motor fiber; sensory fibers only pick up information from an organ and send it to the brain. Thus nerve fibers that cause cardiac slowing must be motor fibers (choices B and D can be eliminated). Specifically, the fibers that reduce the heart rate are parasympathetic fibers (choice A can be eliminated). 147. A. The type of neurotransmitter released is irrelevant (choices B and D, while true, do not answer the question and can be eliminated). The interconnected ganglia of the sympathetic system allow for rapid systemic response, because if one ganglion gets stimulated, it can rapidly stimulate all ganglia in the system. If the ganglia are not interconnected, as in the parasympathetic system, this is not possible (choice D can be eliminated). 148. D. Physostigmine in this case is acting as an “inhibitor of an inhibitor.” Acetylcholinesterase breaks down acetylcholine, thus decreasing its levels. If physostigmine inhibits acetylcholinesterase, then acetylcholine would not be degraded, and its levels would rise (choices A and C can be eliminated). The passage states that the effect of acetylcholine on the pupil is to cause constriction (choice B can be eliminated).

Passage II 149. C. First, glucose is never used directly as an energy source; it is oxidized and the energy released is stored in the form of ATP. In any case, osmosis is a passive process and does not require an energy source (choice A can be eliminated), nor does water cross the plasma membrane through “ion exchange pumps” (choice B can be eliminated). Choice C is true. Additional glucose molecules would raise the osmotic pressure (think “concentration”) of the extracellular space; this in turn would promote the movement of water out of the cell by osmosis (cellular dehydration). Choice D may or may not be true. The passage implies that tissue glucose increases, but whether this occurs by exchange with water molecules is not clear. This makes C a better choice than D. 150. A. Hyperglycemia is high extracellular or plasma glucose levels. Insulin’s job is to reduce plasma glucose levels, so suppressing insulin secretion could lead to higher plasma glucose, and would support the hypothesis. Glucagon has the opposite effect of insulin. Its job is to raise plasma glucose; suppression of this hormone could not lead to hyperglycemia (choice B can be eliminated). Glucose released from the liver into the plasma comes from the catabolism (breakdown) of glycogen. If this process were slowed, it could not cause hyperglycemia (choice C can be eliminated). Lastly, increased sensitivity of all pancreatic endocrine responses would lead to increased levels of both insulin and glucagon in the blood. While excess glucagon could certainly lead to hyperglycemia, excess insulin could not — the hormones have opposite effects (choice D is eliminated).

28

151. A. The question specifically asks for the relationship of a beating heart to the cryoprotective role of glucose; since temperature equilibration and a reduction in the rate of ice formation are not glucose-related, choices B and C can be eliminated on the basis of irrelevance. Choice D is just false; a beating heart requires a constant supply of ATP, not glucose, as an energy source. Choice A is true and has the most relevance with respect to the question being asked. 152. D. The passage states that only extracellular water freezes. Cytoplasm is intracellular fluid, so Item I is false and choice C can be eliminated. Blood plasma is extracellular; Item II is true, and choice B can be eliminated. Lymph is also extracellular; Item III is true, and choice A can be eliminated. 153. D. The frog’s tissues would be forced to rely solely on anaerobic respiration once the heart stopped beating and no longer circulated blood and oxygen throughout the body. Based on the graph of Heart Rate in Figure 1, this occurs between 12 and 24 hours after the onset of freezing. 154. A. Figure 2 shows that frogs injected with glucose had a much higher rate of survival than frogs injected with saline, and that the rate of survival increased with increased glucose. Thus it is clear that glucose has some cryoprotective role in frogs (choices C and D can be eliminated). 155. B. Choices A, B, and D to some extent are true. However, the main “theme” of the passage is that wood frogs can survive freezing episodes by increasing their extracellular glucose concentrations, and that the primary mechanism by which this occurs is accelerated glucose release from glycogen stores in the liver. Therefore, it seems that the most effective physiological condition for surviving freezing episodes would be to have ample liver glycogen stores (choice D is true, but B is better). Choice A might be tempting, since the role of the excess glucose is to cause cellular dehydration, but simple dehydration in the absence of glucose is not described in the passage as a survival mechanism. Choice C is not discussed anywhere in the passage as being a method of surviving freezing.

Passage III 156. D. The passage states that Compound C is soluble in both dilute acid and base. These data suggest that Compound C has both a basic and an acidic functional group. Of the choices given, only D contains both a basic (amine) and an acidic (carboxylic acid) functional group. 157. C. Since all four compounds in Table 1 contain a C=O double bond, they will all have a strong, sharp band at 1700 cm–1 in their IR spectra. 158. B. While pure compounds typically have sharp melting points, impure mixtures tend to melt over a broad temperature range. This eliminates choices C and D. Choice A can be eliminated since the melting point of a mixture is never higher than that of the components of the mixture. 159. C. The passage states that Compound B is a water-soluble alcohol. It is most likely water soluble due to the hydrogen-bonding ability of its hydroxyl group. 160. B. The fact that Compound A slowly dissolves in refluxing aqueous NaOH to result in the formation of two new compounds is consistent only with choice B. An ester is hydrolyzed under such conditions to form a carboxylate and an alkoxide. 161. C. Only choice C, 4-aminobenzoic acid, is an amine. Hippuric acid and diethylbarbituric acid are amides, not amines. 162. D. Since three of the four compounds in Table 1 contain carboxylic acids, they cannot be distinguished based on solubilities or reactivities toward alcohols. Thus, we can eliminate choices A and B. Since all four compounds melt in the range of 183–191°C, they cannot be unambiguously distinguished based on their melting points. They can only be distinguished by their molecular weights.

29

Passage IV 163. C. The passage states that the genes for conjugation are carried on a plasmid (choice A can be eliminated), and that during cell division, plasmids may not be equally distributed among daughter cells. It may be that one of the daughter cells failed to receive the plasmid that carried the genes for conjugation. The cell membrane has nothing to do with moving the replicated chromosomes apart (choice B can be eliminated), and bacteria are prokaryotic and do not contain membranebound organelles like lysosomes (choice D can be eliminated). 164. D. Antibiotics do not lead to mutations in bacteria (choice A can be eliminated), nor can bacteria develop an “immune reaction” to antibiotics. An immune system is required for that, which bacteria do not have! (Choice B can be eliminated.) There is nothing in the passage to support choice C, so it can be eliminated. It is most likely that, due to random mutation, some of the patient’s E. coli were resistant before treatment. Since they were not killed by the antibiotics, they continued to reproduce. 165. A. Of the choices given, the best answer is choice A, although this is still a relatively unlikely possibility. However, antibiotics do not induce mutation (choice B can be eliminated), the rate of reproduction has nothing to do with antibiotic sensitivity (choice C can be eliminated), and modification of metabolism would not alter antibiotic sensitivity (choice D can be eliminated). 166. C. The appendix is found at the beginning of the colon, so if it were to rupture, the E. coli that normally inhabit the colon could enter the abdominal cavity and cause serious problems (choice C is true, and choices B and D are false). M. tuberculosis causes tuberculosis, not appendicitis (choice A can be eliminated). 167. A. The persistence of an organism in any environment is determined primarily by the number of surviving offspring it can produce (A is true). The ability to produce more vitamins (choice B) or to metabolize glucose faster (choice C) do not affect E. coli’s ability to reproduce, nor does their lack of pathogenicity (choice D). 168. B. No digestive enzymes are produced by the bacteria, and no nutrient absorption occurs in the colon (choices A and D can be eliminated). Choices B and C actually are equally likely to occur; the “jobs” of E. coli in the large intestine are to produce vitamins and to reduce the growth of other, pathogenic bacteria. However, since the passage specifically mentions vitamin production and does not mention reduced growth of other bacteria, choice B is a better answer than C.

Independent Questions 169. B. Of the choices given, actin will bind only to myosin molecules. (Don’t be tempted by choice A; during muscle contraction, ATP binds to myosin, not to actin.) 170. C. N-1 has two covalent bonds and two lone pairs, so it is similar to a deprotonated amine and has a (–) charge. N-7 has four covalent bonds and is thus an ammonium ion and has a (+) charge. 171. A. Beginning with the neuron that senses the painful stimulus, the neurons, in order, are a sensory neuron, an interneuron, and a motor neuron. In order to perceive the pain, the sensory information would have to be relayed to the brain and processed. Additional neurons could be placed anywhere around the interneuron (which is found in the central nervous system at the spinal cord level); in other words, at either synapse II or synapse III. 172. C. First, always remember that the phrase “randomly mating population” is a tip off for using Hardy–Weinberg in some way. The question states that 160 members of a 1000-member population exhibit a specific recessive trait. A homozygous recessive genotype is required to exhibit a recessive trait, thus 16% of the population is homozygous recessive. In the equation for genotype frequency, p2 + 2pq + q2 = 1, the value of q2 is 0.16; thus, q = 0.4. Plugging this value for q into the equation for allele frequency, p + q = 1, we find that p = 0.6. The question asks for the number of individuals in the population that are carriers of this trait. Carriers are those individuals heterozygous for the trait; they “carry” the allele, but don’t express it because of their dominant allele. In the equation for genotype frequency, these are the “2pq” individuals. Plugging in the values we obtained previously, we get 2pq = 2 × 0.6 × 0.4 = 0.48, or 48% of the population is made up of carriers. Since 48% of 1000 is 480, the answer is choice C. 30

Passage V 173. A. Blood cells generally have short life spans; if they were not replaced continuously by stem-cell division and differentiation, we would quickly run out of blood cells, and the number of cells remaining would be incompatible with life. Blood cells are stored in the spleen (choice B is false), they do not exit the body through the urinary and digestive systems (choice C is false), and they do not differentiate into other cell types (choice D is false). 174. A. The development of hormone receptors on the cultured stem cells implies that their differentiation will be controlled by the hormones that bind there. More rapid division (choice B) does not address the differentiation issue; in other words, the cells are dividing more rapidly, but is their differentiation random or controlled? Random differentiation supports the stochastic view (choice C can be eliminated), and choice D does not seem to support either the deterministic view or the stochastic view. 175. B. Stem cells that express hormone receptors support the deterministic view, but if the expression of the receptors is random, this also supports the stochastic view. If hormone X causes the formation of erythrocytes, this supports only the deterministic view (choice A can be eliminated). If the receptors are being expressed in response to specific signals, this also supports the deterministic view (choice C can be eliminated). Choice D does not seem to support either view. 176. D. If all blood-cell types need to be replaced, then a totipotent stem cell (one that can differentiate into all cell types) needs to be introduced (choices A and B can be eliminated). Furthermore, the greatest advantage would come from a totipotent stem cell that differentiated deterministically, as this would allow it to produce cell types according to the specific needs of the new host, as directed by the host’s hormones and other external signals (choice C can be eliminated).

Passage VI 177. C. While hydration and oxymercuration–demercuration make the more-substituted alcohol, hydroboration–oxidation makes the less-substituted alcohol. Therefore, choice C is correct. 178. A. After Step 1, the organomercurial alcohols formed have the –OH and the Hg(OAc) groups trans on the ring. This relative stereochemistry is most likely achieved through the intermediate in choice A wherein the 3-membered ring is opened by H2O, as shown below: HgOAc HgOAc

H2O

HgOAc –H

OH2

OH

179. C. This reaction sequence places the –OH group on the less-substituted carbon atom of the C=C double bond, and as stated in the passage, does not involve skeletal rearrangements. 180. D. The O–H stretch of alcohols occurs at 3500 cm–1 in IR spectra. 181. A. Since Items II and III are two-step synthetic reactions, they are not likely to be reversible. Therefore, only choice A can be correct. 182. C. Since the two isomeric organomercurial alcohols in Equation 2 are non-superimposable mirror images of each other, they are enantiomers.

31

Passage VII 183. C. The fact that relaxation due to ACh only occurred in the ring with intact endothelium supports the conclusion that intact endothelium is necessary for this process. The tension increases in both rings upon addition of NE (intact endothelium and damaged endothelium), so this gives us no information about the role of the endothelium specifically, and in any case NE is not ACh! (Choice A can be eliminated.) Tension decrease in both rings is approximately equal, and if anything, is slower in the ring without endothelium (choice B can be eliminated), and of course tension decreases during washout in the ring without endothelium. The NE that caused the increase in tension in the first place is being removed. Still, this gives us no information about the role that the endothelium plays in response to ACh (choice D can be eliminated). 184. C. Since we have no data on the effects of ACh in the absence of NE, we cannot make any conclusions about NE’s effect on the smooth muscle’s sensitivity to ACh (choices A and B can be eliminated). Clearly ACh, which would cause a reduction in tension, has no effect at 10–8 M, so it is untrue to say that its effect is greatest at this concentration (choice D can be eliminated). Since concentrations of ACh above 10–6 M were not tested, and since the ring with endothelium responded to an ACh concentration of 10–7 M, it is fair to say that the presence of endothelium caused at least a 10-fold increase in sensitivity of the aortic muscle. 185. B. Muscle tension should decrease in the presence of ACh. This only occurred in the ring with endothelium; the ring without endothelium was not sensitive at all to ACh at any concentration tested (choice D can be eliminated). Since muscle relaxation occurred at 10–7 M ACh, choice B is the best response. 186. B. The two major factors that determine blood pressure are the cardiac output and the peripheral resistance. L-NMMA is not a naturally-occurring substance; the passage describes it as one of the enzyme inhibitors that “were developed” (choice A can be eliminated). While blood volume does play a role in blood pressure, not all the L-arginine consumed will be converted to NO, so the amount of L-arginine in the diet cannot tell us anything about the status of the blood pressure (choice C can be eliminated). Choice D is tempting because it mentions the two factors that determine cardiac output (heart rate and stroke volume) and we know that cardiac output is a major determinant of blood pressure. However, choice D fails to take into account the other major determinant of blood pressure, the peripheral resistance. 187. A. The somatic nervous system deals only with the stimulation and contraction of skeletal muscle, not blood vessels (choices C and D can be eliminated). Furthermore, since ACh causes vasodilation (a parasympathetic effect) and NE causes vasoconstriction (a sympathetic effect), choice B can be eliminated. 188. D. Vasoconstriction would lead to an increase in blood pressure (choice A can be eliminated) and a reduction in blood flow (choices B and C can be eliminated). It would be most helpful in a situation where blood pressure was rapidly dropping and needed to be increased, such as during a hemorrhage.

Passage VIII 189. A. The bond angle in an equilateral triangle is 60°. 190. A. According to the passage, hydrolysis of an epoxide under acidic conditions results in inversion of stereochemistry. This means that a trans diol must result, eliminating choices C and D. Choice B can be eliminated because it makes no sense; one cannot have a diol that is both axial and equatorial. 191. A. Since both carbon atoms of the epoxide are equally substituted, the regiochemistry is dictated by sterics. Since R is much larger than R′, attack of Nu: will be primarily at the carbon bearing R′. This eliminates choice C. Choice B can be eliminated because the stereochemistry of the left carbon in the product is drawn incorrectly (it is inverted). Choice D can be eliminated because the passage states that the configuration of the carbon that is attacked by the nucleophile undergoes inversion. This leaves choice A as the answer.

32

192. C. A 5-membered ring results from intramolecular hydrogen bonding in a 1,2-diol: H

3

4

..

1

..

O 2

5

H

O

Ph

193. D. Since styrene oxide acts as an electrophile in these ring-opening reactions, the greater nucleophilicity of diethylamine over ethanol explains its higher reactivity.

Independent Questions 194. A. CBr4 is the least polar of the bromomethanes because the four C–Br bond dipoles cancel each other out due to the tetrahedral symmetry of the molecule. This results in no dipole whatsoever for CBr4. 195. C. Tautomerism is a structural equilibrium and usually involves movement of a proton, as in this keto–enol equilibrium. 196. B. Remember the basic themes about fungi: they have a haploid life cycle (choice A can be eliminated), they possess a cell wall (choice D can be eliminated), and they can reproduce asexually by spore formation. Spores are, in a sense, like seeds. They are surrounded by a tough coat that helps them survive environmental extremes. When they have “weathered the tough times” and are again in a favorable environment, they will germinate (choice C can be eliminated). 197. B. The heart is derived from mesoderm, and so is the skeletal system. The eye and spinal cord are derived from ectoderm (choices A and C can be eliminated), and the liver is derived from endoderm. 198. A. A man with normal blood clotting has the genotype XY. Because women receive an X chromosome from their fathers, a woman with a hemophiliac father must carry at least one X chromosome with the recessive allele for hemophilia on it. However, because she displays normal blood clotting, her other X chromosome must be normal; in other words, she is heterozygous and has the genotype XhX. Here’s the Punnett square: Xh

X

X

X h X XX

Y

X h Y XY

From the Punnett square we can see that the probability of having a son with hemophilia (genotype XhY) is 1/2. For the probability of all three of their sons having hemophilia, use the Rule of Multiplication: 1/2 × 1/2 × 1/2 = 1/8.

33

Passage IX 199. D. Since T cells are not discussed in the passage anywhere, and since the antibody binds to a macrophage protein, choices B and C can be eliminated. From the diagram, it is clear that the events leading to septic shock involve activation of macrophages. If this is to be prevented, macrophage activation must be inhibited (choice A can be eliminated). 200. A. Drugs that decrease inflammation would not affect platelet or red blood cell counts (choices B and C can be eliminated). The inflammatory response depicted in the diagram shows hypotension, a decrease in blood pressure, as part of the events leading to shock. However, blocking the inflammatory response would not necessarily lead to an increase in blood pressure, it would merely prevent the decrease associated with this cascade of events (choice D can be eliminated). (Note that while choice A is not obviously true based on the passage and requires some outside knowledge to answer, this question should be tackled using process of elimination, as the other three choices are clearly incorrect.) 201. C. If the blood is pooling in the venous circulation, then less of it is returning to the heart. Decreased venous return leads to decreased stroke volume, which leads to decreased cardiac output, which leads to decreased blood pressure. Thus, choice B would occur and can be eliminated. Because of the decreased cardiac output (and the venous pooling), less blood is being passed through the pulmonary system, so the blood overall will have a lower oxygen content (choice D is true and eliminated). Lastly, if oxygen content is decreased, cells will rely more heavily on anaerobic respiration, the end-product of which is lactic acid (choice A is true and eliminated). Only choice C would not occur. Venous pooling would lead to edema as fluid moved from the veins to the surrounding tissues, and in an attempt to compensate, lymphatic fluid volume, if anything, would increase. 202. A. Fever (increased body temperature) would lead to dilation of capillaries in the skin in an effort to direct more blood to the surface of the body so as to increase heat loss. Increased skeletal muscle activity (i.e., shivering) and decreased respiration rate are compensatory mechanisms for when the body temperature is too low (choices B and C can be eliminated). If anything, fever would result in increased fluid loss in an attempt to cool the body through perspiration (choice D can be eliminated). 203. C. Since DNA replication occurs in S phase, a drug that interferes with DNA replication would cause cells to arrest in this phase.

Passage X 204. B. AlCl3 is quite a strong Lewis acid since the aluminum atom has only a sextet of electrons. In electrophilic aromatic substitutions, a catalyst such as AlCl3 is usually used. 205. D. Since a new group is added (alkyl), and an old group is removed (H), this is termed a substitution, not an addition. Thus, choices A and B can be eliminated. Since the incoming group is an electrophile (n-propyl chloride), this is an electrophilic aromatic substitution. 206. D. Choices B and C can be eliminated since the molecules contain no stereocenters. Choice A can be eliminated since the molecules do not differ in σ-bond rotations. The molecules are structural (constitutional) isomers because they have different structures. 207. C. Molecule 1 will elute first since it has the lowest molecular weight, and molecules elute on GC approximately according to increasing boiling point. This eliminates choices B and D. Since branched isomers boil at lower temperatures than their straight-chain isomers, choice C is a better response than choice A.

34

Passage XI 208. B. The best evidence that something is required to activate a pathway is to show that the pathway cannot be activated in the absence of that something. Therefore, choice B clearly shows that G proteins are required to regulate intracellular signaling. Choice A is good evidence, but not as good as choice B. Choice C is extremely weak evidence; if the ratios are not even consistent from cell type to cell type, how can it be said that G proteins and adenylate cyclase play any role together in signaling pathways? Lastly, choice D presents evidence for the fact that G proteins may not play a role in intracellular signaling, since the pathway can be activated in their absence. 209. D. Choices A, B, and C are all described in the second paragraph as being part of the chain of events involved in intracellular signaling. Choice D, however, is not mentioned anywhere, and what would we get if GTP were phosphorylated anyway? GQP (guanosine quadrophosphate)? 210. D. Binding of an activated G protein to an inactive enzyme is unlikely to change the primary structure (i.e., the amino acid sequence) of either player involved (choices A and B can be eliminated). Furthermore, it can be assumed from the passage that binding of an activated G protein to an inactive enzyme would activate that enzyme. Remember that the two most common ways to activate (or for that matter, to deactivate) an enzyme is to (1) phosphorylate it, or (2) change its shape (tertiary structure). Since the enzyme is clearly not being phosphorylated, the best answer here is choice D. Note that it is unlikely that binding of the G protein to the inactive enzyme would have any effect on the structure of the G protein itself; the point is to change the enzyme, not the G protein (choice C can be eliminated). 211. A. As in Question 208, choice D, if the enzyme can be activated in the absence of a G protein, this would clearly refute the hypothesis that the G protein is required. If choice B were true, this would present weak evidence for the necessity of the signaling pathway; and if choice C were true, this would present even better evidence for the role of the G protein in this pathway. If the enzyme were always found in the activated state (choice D), this would help refute the hypothesis, but this is not as good as choice A. 212. B. This is another question that is best tackled by process of elimination. The GTP is not bound to the protein until after it binds to the receptor (choice A is false). Hydrolysis of GTP does not occur until the end of the pathway, well after the activation of both the G protein and the membrane-bound enzyme (choices C and D are false). In fact, hydrolysis of GTP serves to inactivate the G protein. While choice B is not explicitly stated in the passage, it seems a likely method, and with the other three choices being false, it is the best answer choice. 213. C. This is specifically stated near the end of the second paragraph (choice C is true and choice A is false). Choice B may be true, but does not deal with the question being asked, which assumes that the G protein has already been activated. Lastly, choice D is simply false. Binding of the G proteins to the membrane-bound enzyme does not result in hydrolysis of GDP.

Independent Questions 214. D. Diffusion of gases is not rapid enough to inflate the lungs (choice A can be eliminated), and respiratory gases do not need to be actively transported, because they are lipid soluble (choice B can be eliminated). The lungs are inflated when the diaphragm contracts, increasing the size of the chest cavity and creating negative pressure; air then rushes in to fill this “vacuum” (choice C can be eliminated). 215. A. Bacteria reproduce by fission, but viruses do not. Bacteria have a rigid cell wall but are not the only organisms that do; plants also have a cell wall, so the presence of this structure would not necessarily classify the organism as a bacterium (choice B can be eliminated). Bacteria do not have a nuclear membrane (choice C can be eliminated), and some viruses contain both RNA and protein (enzymes, choice D can be eliminated).

35

216. A. The function of aldosterone is to increase Na+ reabsorption by the kidneys (choices B and D can be eliminated), in other words, to increase the amount of Na+ retained by the body (remember that reabsorption means to take something from the urine and return it to the bloodstream). If excessive quantities of NaCl were consumed, there would be no reason to increase reabsorption of Na+; if anything, the level of aldosterone would decrease (choice C can be eliminated). 217. A. Only Molecule 1 is not an ether (R–O–R). Molecule 1 is an ester (RO(CO)R). 218. B. The thyroid gland and the adrenal glands have no role in the menstrual cycle (choices A, C, and D can be eliminated). The hypothalamus releases GnRH, which stimulates the anterior pituitary to release FSH and LH, which stimulate the ovary to undergo oogenesis and follicle maturation, as well as the secretion of estrogen and progesterone, which regulate the uterus during the menstrual cycle. 219. D. A cross between homozygous long white plants (LLWW) and homozygous short red plants (llRR) would produce an F1 generation that was heterozygous at both locations (LlWR). These plants would be pink and have long leaves. If they were bred randomly, all phenotypes would appear. There are two leaf phenotypes (long and short) and three color phenotypes (white, pink, and red), and thus 2 × 3 = 6 total phenotypes: long white, long pink, long red, short white, short pink, and short red.

36

View more...

Comments

Copyright ©2017 KUPDF Inc.
SUPPORT KUPDF